You are on page 1of 224

2014-2015

This book focuses on the high yield essential content for


the most commonly encountered problems in the exam

PROVEN, PRACTICAL TOOLS TO HELP YOU PASS THE EXAM

Abdulla Ali Isa, MBBS

Content
1- Preface
2- General information about BMLE
3- Cardiology
4- Respiratory
5- Infectious diseases
6- Gastroenterology
7- Hematology and Oncology
8-Nephrology
9- Endocrinology
10- Rheumatology
11- Neuroscience
12- Pediatrics
13- Obstetrics and Gynecology
14- Surgery
15- Orthopedic surgery
16- Otorhinoloryngology
17- Ophthalmology
18- Family medicine
19- Emergency medicine
20- Psychiatry
21- Dermatology and Immunology
22- Objective Structured Clinical Examination

Preface

Preparing for and doing well on the BMLE are essential requirements on
the road to becoming a practicing physician in Kingdom of Bahrain. This
book have been designed to be a great memory aid and last minute
review so that you have the key facts required to pass the exam.
The material has been selected based on my own experience revising
and taking BMLE. I have written this book in order to provide the
maximum amount of information with the minimum words and help you
in the decision making process.
The logical arrangement of medical facts will make it possible to review
most common BMLE subjects just a few days before the exam. I believe
that this approach will be most helpful for final year medical students
and interns.

Best luck on your BMLE.


Abdulla Ali Isa
Abdullaali10@live.com

General information about BMLE


What is BMLE?
-BMLE is Bahrain Medical Licensure Exam.
Who is responsible for organizing the exam?
-The National Health Regulatory Authority.
Importance of BMLE
-It is the first step for licensing of the graduated Bahraini
physicians to practice medical specialty in the Ministry of
Health and other governmental and non-governmental
health services in the Kingdom of Bahrain. So, It is the
way that transfers you from Intern into
doctor/Physician.
Examination committee
-The Examination committee consists of Consultants
from the Ministry of Health specialized in medical
education and representing major disciplines in primary
and secondary healthcare. But a consultant family
medicine is the essential member and responsible for
setting the exam.
Examination content
Multiple Choice Questions (MCQs)
Objective Structured Clinical Examination (OSCE)
Multiple Choice Questions (MCQs);
The exam takes 3 hours (180 minutes).
Subject
Estimated number of
questions
Internal medicine
50
Obstetrics and
30
gynecology
Pediatrics
30
Family medicine
15
Miscellaneous
15
(Ophthalmology, ENT,
Dermatology, Psychiatry)
Surgery
10

Objective Structured Clinical Examination (OSCE);


Each station takes 5 minutes.

Scoring

-This exam constituted of 20 various clinical stations


including;
-ECG, Dummies, Slides, Photos, Laboratory reports,
Calculation, Radiographs, Tools, etc
The scoring of written paper (MCQs) is done by the
computer and all questions are given equivalent credit
point.
The OSCE marking is done by a panel of examiners.
The written exam carries 60% of the total and the OSCE
is 40 %. Candidates should score 50 % or above in the
written exam to be eligible to sit for the OSCE. The final
passing score is the cumulative of 60 % or above from
both written and OSCE examinations.

Cardiology

The most common


Question
Causative organism of subacute bacterial endocarditis
Causative organism of myocarditis
Cause of constrictive pericarditis in developing countries
Cause of heart failure
Cause of Ischemic heart disease
Complication of chronic atrial fibrillation
Calcium channel blocker causing constipation
Cause of left ventricular hypertrophy
Modifiable risk factor for cardiovascular disease (CVD)
Cause of secondary hypertension

1
2
3
4
5
6
7
8
9
10
11
12
13
14
15
16

Cause of acute infectious endocarditis worldwide


Cause of right heart failure in adults
Cause of death in a patient with MI during the first 72 hours
Cause of death in a patient with MI after the first 72 hours
Type of congenital defect of the aortic valve
Characteristic manifestation of a floppy mitral valve

Type of shock
Cardiogenic
Septic
Neurogenic
Hypovolemic

Answer
Viridans Streptococci
Coxsackievirus
Tuberculosis
Ischemic heart disease
Atherosclerotic disease
Cerebrovascular accident
Verapamil
Hypertension
Essential hypertension
Renal artery stenosis
Staphylococcus aureus
Left heart failure

Arrhythmia
Myocardial re-infarction
Bicuspid aortic valve
Midsystolic click

Most common cause


Myocardial Infarction
E.Coli and Staphylococcus aureus
Spinal cord injury in the cervical or thoracic region
Massive hemorrhage

Diagnostic or radiological tool

The most appropriate next step in diagnosis of a patient presented with a history suggestive of
acute bacterial endocarditis is Echocardiogram.
Ergonovine echocardiography before coronary angiography is safe and can be used as a reliable
diagnostic screening test for coronary vasospasm in patients with negative treadmill or normal
stress myocardial perfusion scan results.
Pre-CABG carotid Doppler is routinely recommended, because concomitant carotid disease is
often associated with CAD.
The best predictor of sudden cardiac death is an ejection fraction 35%.

The most appropriate next step in management of a patient diagnosed to have


streptococcal bovis endocarditis with vegetation is colonoscopy, because it is associated
with colonic pathology ranging from diverticuli to polyps to colon cancer.
Vasovagal type of syncope can be tested by tilt table testing which will demonstrate
hypotension and bradycardia.

Laboratory markers
Brain natriuretic peptide (BNP)
Troponin

A marker for Heart failure and helps to distinguish congestive


heart failure from pulmonary pathology as the etiology of
dyspnea
The Preferred marker for detecting myocardial cell injury

Risk factors

Question
The most important risk factor for stroke

Answer
Hypertension

The greatest risk for stroke in a patient with atrial


fibrillation

Previous history of stroke

Medical signs

Question
The most supportive sign of Congestive Heart
Failure
Cardiogenic shock is associated with

The most reliable sign of left heart failure

Answer
Raised jugular venous pressure
(JVP)
Raised jugular venous pressure
(JVP)
S3 heart sound

Coronary Artery disease


Most dangerous risk factor
Elevated low density lipoprotein
Most common risk factor
Hypertension
Most likely beneficial measure to decrease the risk
Regular exercise

Heart disease
Effect of Valsalva
maneuver

-ACEIs are
contraindicated in
renal artery stenosis,
pregnancy and in a
patient with previous
history of
angioneurotic edema.

Hypertrophic obstructive
cardiomyopathy
The murmur increases in
loudness.

Mitral valve prolapse

Most murmurs

The murmur of
becomes longer, and
may also become
louder.

Decrease in length and


intensity.

Angiotensin converting enzyme inhibitor


-ACE inhibitor-The best agent used
induced cough is
to decrease mortality
thought to be
rate in patients with
mediated by
congestive heart
bradykinin and
failure, and diabetic
substance p.
nephropathy is ACEI.

-ACEI is not useful in


treatment of Angina
pectoris.

Electrolyte Imbalance
Hyperkalemia

Hypokalemia
Hypercalcemia
Hypocalcaemia
Hypomagnesaemia

Anatomical location
Anteroseptal wall
(Worst prognosis)
Lateral wall

ECG findings
Tall peaked T waves (first sign); a wide QRS; PR prolongation; and loss of
p waves.
T-wave flattening, U waves, and ST-segment depression
short QT interval
prolonged QT interval and ST segment
Prolonged PR and QT interval

Myocardial infarction
Coronary Artery
Left anterior descending coronary artery

ECG findings (ST changes)


ST elevation in leads V1-V4

Left Circumflex coronary artery

ST elevation in Leads I, aVL, and V5- V6

Left Circumflex coronary artery


Right coronary artery

ST depression in leads V1-V2


ST elevation in II, III and aVF

Posterior wall
Inferior wall

Mitral stenosis
Mitral regurgitation
Mitral valve prolapse
Hypertrophic cardiomyopathy
Patent ductus arteriosus
Atrial septal defect
Ventricular septal defect
Aortic regurgitation
Aortic stenosis

Tricuspid regurgitation
Tricuspid stenosis

The murmurs
Diastolic rumble with opening snap.
Holosystolic blowing murmur radiating to axilla.
Mid-systolic click.
Systolic, brisk upstroke, parasternal heave.
Continuous, machinery murmur.
Fixed, split S2.
Holosystolic murmur radiating to the right lower sternal border.
Water-hammer pulse, decrescendo mid-diastolic.
Harsh, crescendo/decrescendo systolic murmur radiating to
carotids.
Holosystolic murmur radiating to the left upper sternal border.
Diastolic decrescendo murmur

Cardiomyopathy
Dilated (Congestive)
Hypertrophic
Dilated ventricle,
Ventricular
normal wall thickness.
hypertrophy.
Left and right
Small cavity.
ventricular failure.
Septum may obstruct
outflow.
Caused by:
Caused by:
o Alcohol (Most
o Often genetic
common)
Most cases are
o Doxorubicin
autosomal dominant.
o Infection (usually
viral)

Restrictive
Low ventricular
compliance.
(Restricts diastolic filling)

Caused by:
o Amyloidosis
o Sarcoidosis
o Hemochromatosis

Drugs

Metoprolol
Metoprolol
Beta blockers
Beta blockers
Digoxin

Heparin

Aspirin

NSAIDs

Atorvastatin (Lipitor)

Fludrocortisones,
Midodrine, and
Physostigmine

Epinephrine
Norepinephrine

Cardiovascular pharmacology
Comment
Cardioselective drug that could be used in patients with COPD.
Used as an Anti-Arrhythmic drug post myocardial infarction.
Primary treatments for symptomatic mitral valve prolapse.

Contraindicated in treating cocaine induced arrhythmias.


Contraindicated in Hypertrophic obstructive cardiomyopathy.
First step in management of anaphylactic reaction.
Recommended first-line vasopressor agent to correct hypotension in
patients with sepsis.
The most effective proven treatment to prevent progression of unstable
angina to myocardial infarction.
The only treatment that has been shown to reduce postoperative
cerebrovascular events in patients undergoing coronary artery bypass
grafting in the first 48 hours after surgery.

Should be avoided - If possible- in patients with heart failure,


because they cause sodium and water retention, as well as an
increase in systemic vascular resistance which may lead to cardiac
decomposition.
An antihyperlipidemic agent that provides greater reduction in LDL
levels.
Effective treatments for chronic orthostatic hypotension.

Wolff-Parkinson-White syndrome
Treatment
Drugs must be avoided
Procainamide (treatment of choice)
Adenosine
Amiodarone
Digoxin
Cardioversion
Calcium channel antagonists
A destruction of the abnormal
Beta blockers
electrical pathway by radiofrequency
catheter ablation (definitive treatment)

Classic clinical clues that suggest a diagnosis of renal-artery stenosis


The onset of stage 2 hypertension (blood pressure >160/100 mm Hg) after 50 years of
age or in the absence of a family history of hypertension.
Hypertension associated with renal insufficiency, especially if renal function worsens
after the administration of an agent that blocks the renin-angiotensin-aldosterone
system.
Hypertension with repeated hospital admissions for heart failure.
Drug-resistant hypertension (defined as blood pressure above the goal despite treatment
with three drugs of different classes at optimal doses).

Definitions

Question
The most probable diagnosis of a patient presented with high BP,
radiofemoral delay, left ventricular hypertrophy on ECG, and (3
figure sign), cardiomegaly and rib notching on CXR
The most probable diagnosis of a patient presented with high
blood pressure and unequal pulse
Acute myocardial damage most often occurring in post
menopausal women immediately following an overwhelming
emotionally stressful event and it leads to ballooning of left
ventricular apex and dyskinesis. Its mechanism involved massive
catecholamine discharge.
A collection of three medical signs associated with acute cardiac
tamponade, an emergency condition wherein fluid accumulates
around the heart and impairs its ability to pump blood. The signs
are low arterial blood pressure, distended neck veins, and distant,
muffled heart sounds.
A type of angina is more common in female than male and it could
present with ST segment elevation more than ST segment
depression.

Answer
Coartication of
Aorta
Aortic dissection
Tako-Tsub
cardiomyopathy

Beck's triad

Prinzmetal angina

CHA2DS2-VASc: It is a clinical prediction rule for estimating the risk of stroke in patients with nonrheumatic atrial fibrillation (AF), a common and serious heart arrhythmia associated with
thromboembolic stroke. It is used to determine whether or not treatment is required with
anticoagulation therapy or antiplatelet therapy.
Risk factors:
- Congestive heart failure (or Left ventricular systolic dysfunction)
-Hypertension
- Age 75 years
- Diabetes Mellitus
-Prior Stroke or TIA or thromboembolism
- Vascular disease (e.g. peripheral artery disease, myocardial infarction, aortic plaque)
- Age 6574 years
- Sex category (i.e. female sex)
Treatment of a patient with no/low risk factors Treatment of a patient with Moderate or High
risk factors
Aspirin
Oral anticoagulant with a goal INR of 2.0-3.0

Apoprotein A1
It is inversely related to cardiovascular disease risk.
Levels are increased in nephritic syndrome.
It is associated with high density lipoprotein.
It is involved in removal of cholesterol by HDL from tissues and facilitates transfer back to
VLDL and the liver. It is not a component of chylomicrons.

What are the types of


arrhythmia that often
disappear with
excretion?
-Premature
ventricular
contractions (PVC).
-Premature atrial
contractions (PAC).

Questions and answers


What are the
What is the
indicators of carditis
recommended
in a patient with
Antibiotic Regimen for
rheumatic heart
Infective endocarditis
disease?
Prophylaxis?
-Presence of diastolic -Amoxicillin 2 gram
or long systolic
orally 1 hour before
murmurs.
each procedure.
-Pericarditis.
-Cardiomegaly.

What are the effects


of Dobutamine and
Dopamine?

-Dobutamine increases
inotropy without
vasoconstriction.
- Dopamine increases
inotropy with
vasoconstriction.

Notes

Respiratory

The most common


Question
1 Causative organism of typical community-acquired pneumonia
2 Causative organism of atypical community-acquired pneumonia
3 Causative organism of Hospital-acquired pneumonia

4 Causative organism of community-acquired pneumonia in


chronic alcoholics
5 Nursing homeacquired pneumonia

Gram-negative diplococci
Gram-positive diplococci in chains
Gram-positive cocci in clusters
Gram-negative rods
Gram-positive rods

Answer
Streptococcus pneumoniae
Mycoplasma pneumoniae
Gram-negative bacilli such as E.Coli
and Pseudomonas, Enterobacter,
Klebsiella, Proteus
Klebsiella pneumonia
Methicillin-resistant
Staphylococcus aureus (MRSA) and
Pseudomonas Aeruginosa

Respiratory Infections
Moraxella catarrhalis
Streptococcus pneumoniae
Staphylococcus aureus
Corynebacterium diphtheria
-Klebsiella
-Enterobacter
-Proteus

Radiology

The most appropriate study to perform to distinguish between pleural fluid and pleural thickening is
decubitus chest radiograph.
Chest X-ray of incomplete (ball-valve type) obstruction of main bronchus will show hyperinflation of
ipsilateral lung on expiration.

Pulmonary edema is associated with frothy sputum. Chest x-ray showed Bat-wing appearance.

Pharmacology
The primary treatment of pulmonary Embolism is anticoagulant.
The most appropriate empiric therapy for nursing homeacquired pneumonia in a patient with
no other underlying disease is Levofloxacin.

Patients with carbon monoxide poisoning should be treated immediately with normobaric oxygen, which
speeds up the excretion of carbon monoxide.
The most likely drug that causes hypoxemia due to large V/Q mismatch (loss of hypoxic pulmonary
vasoconstriction) is Nitroprusside.

Obstructive pulmonary disease


-FEV1/FVC <75%

Restrictive pulmonary disease


-FEV1/FVC 75%

-COPD (irreversible)
-Asthma (reversible)

-Sarcoidosis
-Pigeon Fanciers lung
-Extrinsic allergic alveolitis

How to differentiate between COPD and Asthma?


-Through Bronchodilator challenge
Asthma
COPD
-Increase in FEV1
-No change in FEV1

How to differentiate between chest wall weakness and Interstitial lung disease?
-Through Carbon Monoxide Diffusing Capacity (DLCO)
chest wall weakness
Interstitial lung disease
-Normal DLCO
-Decrease in DLCO

Asthma
-The best initial tests in acute asthmatic exacerbation
-ABG
-The peak expiratory flow (PEF)
-Symptomatic indicator
-Increase in FEV1 of more than 12% with the use of
Albuterol
-The best indicator of severity of Asthma
-Respiratory rate
-The single most accurate diagnostic test of stable
- A 20% decrease in FEV1 with the use of
asymptomatic asthmatic patient
Methacholine or histamine.

Asthmatic drugs that must used as a combination


therapy only
Long-acting B2 agonists

Asthmatic drugs that must used both monotherapy


and combination therapy

Inhaled corticosteroids
Leukotriene-receptor antagonists
Short-acting B2 agonists
Mast cell stabilizers

-Recommended monotherapy for symptomatic COPD patients with FEV1 less than 60% is either :
-Long acting anticholinergic or
-Long acting B- agonist e.g. Salemetrol
-Plus a rescue therapy with a short acting B agonist

Asthma
Chronic inflammatory disorder of the airways.
Episodic or chronic symptoms of airflow obstruction; breathlessness, cough, wheezing, and chest tightness.
Symptoms frequently worse at night or in the early morning.
Prolonged expiration and diffuse wheezes on physical examination.
Limitation of airflow on pulmonary function testing or positive bronchoprovocation challenge.
Complete or partial reversibility of airflow obstruction, either spontaneously or following bronchodilator
therapy.
Type
Definition
Medications
Mild intermittent
No
daily
medication
symptoms 2 times a week.
Asymptomatic and normal Peak expiratory
Flow between exacerbations.
Exacerbations brief (few hours to few
days); intensity may vary.
Nighttime symptoms 2 times a month.
Mild persistent
Low dose inhaled corticosteroid
Symptoms > 2 times a week but < 1 time a
or
day
Cromolyn/nedocromil
Exacerbation may affect activity
or
Nighttime symptoms > 2 times a month
Theophylline/leukotriene
modifier
Moderate
Low-medium dose inhaled
Daily symptoms
persistent
corticosteroid
Daily use of inhaled short-acting B2
and
agonist
Long acting bronchodilator
Exacerbation affect activity
(long-acting inhaled B2 agonist,
Exacerbation 2 times a week; may last
sustained-release theophylline
days
or long acting B2 agonist
Nighttime symptoms > 1 time a week
tablets)
Severe persistent
Continual symptoms
High dose inhaled
corticosteroid
Limited physical activity
and
Frequent exacerbation
Long acting bronchodilator
Frequent nighttime symptoms
(long-acting inhaled B2 agonist,
sustained-release theophylline
or long acting B2 agonist
tablets)
and
Corticosteroid tablets or syrup
o In quick relief medication, we used short acting inhaled B2 agonists as needed in mild intermittent type and
on daily basis in other types.
o Long-acting B2 agonist must not be used as monotherapy.

COPD
The best diagnostic test of COPD is Spirometry.
The most common organism associated with acute exacerbation of chronic bronchitis is
H.Influenza.
The most common etiology of COPD is cigarette smoking.
Inhaled corticosteroids will not reduce mortality or affect long-term progression of COPD.
However, they do reduce the number of exacerbations and the rate of decline in the quality of
life. These agents do have side effects, including candidal infection of the oropharynx,
hoarseness, and an increased risk of developing pneumonia.
The most important task that will increase survival in COPD patient is smoking cessation.
Home oxygen increases the survival of patients with COPD.
Sleep disturbance
Most (85%) insomnias are secondary; physiological, psychiatric, physical and drug-induced.
Alcohol decreases total sleep time and the time required to fall asleep.
Narcolepsy is associated with an irresistible urge to sleep and usually presents in the first to
second decade.
Hypersomnia consists of hangover symptoms caused by excessive sleep, which leads to the
need for more sleep.
The most likely diagnosis of an obese male presented with a history of sleeping in daytime and
snoring at night is obstructive sleep apnea syndrome. Sleep apnea is known to be associated with
hypertension.
Shift work insomnia is a circadian sleep disorder and it may respond to bright-light therapy.
Pulmonary hypertension

Definition

Most common cause

A mean pulmonary artery pressure> 25 mmHg at rest or 30 mmHg with


exercise.
Respiratory failure due to intrinsic pulmonary disease.

Light's criteria(suggests exudates)


Pleural fluid protein / serum protein
ratio is greater than 0.5
Pleural fluid LDH / serum LDH ratio is
greater than 0.6
Pleural fluid LDH is greater than twothirds of the upper limit of normal
serum LDH value.

Causes of exudate
-Infection
-Malignancy
-Connective tissue disease
-Chylothorax
-Pancreatitis
-Postcardiotomy syndrome
-Drug-induced(e.g.amiodarone)
-Esophageal rupture
-Uremia
-Subdiaphragmatic abscess

Causes of transudate
-Congestive heart failure
-Nephrotic syndrome
-Cirrhosis
-Hypoalbuminemia
-Urinothorax
-Peritoneal dialysis
-Early atelectasis

Pneumoconiosis
Pneumoconiosis results from inhalation of organic dust (e.g., asbestos, silica and metals).
Asbestos exposure is associated with increased incidence of bronchogenic carcinoma and
mesotheliomas, especially if the patient smokes.
Often, the disease has a prolonged latency period after exposure.
Silicosis is associated with superinfection by Mycobacterium tuberculosis.

Hypersensitivity pneumonitis
Hypersensitivity pneumonitis occurs in persons who have an abnormal sensitivity to an organic agent.
Examples Farmers lung
Caused by exposure to Actinomyces in moldy hay
Pigeon-Breeders lung Caused by exposure to animal protein in bird droppings
Treatment of pneumothorax
A small pneumothorax in a symptomatic patient often requires no therapy.
Pneumothorax >20% usually require treatment.
(a) Tube thoracostomy is the treatment of choice.
(b) Patients who have a history of recurrence may benefit from pleurodesis with intrapleural tetracycline.
Patients who have a tension pneumothorax require immediate therapy with rapid needle
thoracocentesis immediately followed by the insertion of a chest tube.
First line therapy for TB
Isoniaziad
Rifampicin
Pyrazinamide
Ethambutol
Streptomycin

Side effects
Peripheral neuropathy, hepatitis, skin rash, mild CNS effects
Hepatitis, fever, skin rash, flu-like symptoms, GI upset, bleeding problem,
renal failure, orange colored secretion, Leukopenia and thrombocytopenia.
Hyperuricemia
Optic neuritis
8th nerve damage, nephrotoxicity
Quick revision

Adenocarcinoma of the lung is not associated with hypercalcemia.

The most likely diagnosis of a patient admitted as a case of acute pancreatitis and developed progressive
respiratory distress within 12-48 hours is acute respiratory distress syndrome.

Pulmonary arteriovenous malformations are found in 15%30% of patients with hereditary


hemorrhagic telangiectasia (HHT), also known as Osler-Weber-Rendu syndrome.
Klebsiella pneumoniae is associated with hemoptysis from necrotizing disease and "currant jelly"
sputum.
Legionella is associated with gastrointestinal symptoms (abdominal pain, diarrhea) or CNS
symptoms (headache, confusion).
Anaerobes are associated with Foul-Smelling sputum, "rotten eggs".
Pneumocystis is associated with AIDS with <200 CD4 cells.

Small cell carcinoma does not cause clubbing.

Sarcoidosis (non-caseating granuloma)


Grade
CXR
Grade 0
Normal CXR
Grade 1
Lymph node enlargement
Grade 2A
Lymph node enlargement + Diffuse pulmonary parenchymal disease
Grade 2B
Diffuse pulmonary parenchymal disease
Grade 3
Pulmonary fibrosis (honeycombing)
-The most appropriate therapy for pulmonary sarcoidosis is glucocorticoids.

Notes

Infectious diseases

The most common

Question
Cause of an elevated lymphocyte count
Infection transmitted person-to-person in
wrestlers
Causative organism in urinary tract infection,
acute prostitis and epididymitis
Cause of secondary enuresis
Causative organism of Malignant external otitis
Opportunistic bacterial infection in AIDS
patients
Parasitic causative organism for subacute and
chronic diarrhea in AIDS patients
Causes of intracranial masses in AIDS patients

9
10
11
12

Cause of dysphagia in AIDS patients


Cause of death in AIDS patients
Cause of Fever of unknown origin
Causative organism in catheter-related sepsis

1
2
3
4
5
6
7

13 Sexually transmitted disease


14 Bacterial sexually transmitted disease
15 Associated infection with Reiters syndrome
16 Bacterial causative organism of meningitis in
neonates
17 Bacterial causative organism of meningitis in
children (1 month-3years)
18 Bacterial causative organism of meningitis in
children older than 3 years
19 Bacterial causative organism of meningitis in a
child with no history of immunization
20 Bacterial causative organism of meningitis in
adults
21 Bacterial causative organism of meningitis in
immunosuppressed

Answer
Viral infections
Herpes gladiatorum caused by the herpes simplex
virus
Escherichia coli
Urinary tract infection
Pseudomonas Aeruginosa
Mycobacterium avium complex (MAC)
Cryptosporidium parvum
Cerebral toxoplasmosis
Primary brain lymphoma
Candidiasis
Pneumocystis pneumonia
Infectious
staphylococcus aureus
staphylococcus epidermidis
Genital warts
Chlamydia
Chlamydia
Group B Streptococcus
Streptococcus pneumoniae
Neisseria meningitidis
Haemophilus influenzae
Streptococcus pneumoniae
Listeria monocytogenes

Causative organism
Chlamydia trachomatis

Treatment

Erythromycin/Tetracycline

Neisseria gonorrhea
Trichomonas vaginalis
Bacterial vaginosis
Treponema pallidum

Ceftriaxone
Metronidazole
Metronidazole
Penicillin

Sexually transmitted diseases treatment guidelines/Urethritis


History
Intermittent urethral discharge accompanied by Dysuria+ acquiring a new sexual partner.
Treatment Azithromycin, 1 g orally as a single dose or Doxycycline, 100 mg orally twice a day for 7 days.
plus
Ceftriaxone (Rocephin), 250 mg intramuscularly or Cefixime 400mg orally in a single dose.
History
Persisting of the same complaint without acquiring a new sexual partner.
Treatment Azithromycin, 500 mg orally once daily for 5 days, or Doxycycline, 100 mg orally twice daily
for 7 days.
plus
Metronidazole, 2 g orally as a single dose.

PPD 5 mm or greater consider


Positive in:
-HIV Infection
-Recent Tuberculosis contact
-Immunosuppressed (e.g. HIV,
Prednisone >15 mg/day for >1
month, Immunosuppressant)
-Apical fibronodular changes on
Chest X-ray (old Tb scarring) or
other findings on Chest X-ray

Mantoux tuberculin skin test


PPD 10 mm or greater consider
Positive in:
-Health care workers
-New immigrant within last 5
years from developing nations
-Intravenous Drug Abuse
-Children under age 4 years old,
or children/teens exposed to
high risk adults
-Malnutrition
-Diabetes Mellitus
-Cancer
-Chronic Kidney Disease
-Body weight >10% below Ideal
Weight
-Silicosis
-Tuberculosis endemic to region
-High risk living environments
(prison, Nursing Homes,
hospitals, homeless shelter)

Arbovirus infections which are transmitted to


human by mosquito bites
- Japanese encephalitis.
- St. Louis encephalitis.
- Murray valley encephalitis.

PPD 15 mm or greater consider


Positive in:
-All persons (low risk for TB
disease)

Arbovirus infections which are transmitted to


human by ticks
- Louping ill.
- Crimean-Congo hemorrhagic fever.

Characteristic features of HIV seroconversion illness


Maculopapular rash, oral ulceration and generalized lymphadenopathy
Aseptic meningitis
Associated with a poorer prognosis
Positive HIV P24 antigen

In Kaposis sarcoma occurring in AIDS patient


There is preponderance in homosexual males.
There is early involvement of lymph node and viscera.
Visceral Kaposis sarcoma carries a worse prognosis than that confined to skin.
A diffuse pneumonitis may occur.
There is evidence of a genetic predisposition. There is a higher prevalence of HLA-DR5 in these
patients than in the general population.
Human herpes virus 8 is involved in pathogenesis.

Causative
organism
-S.mansoni
-S.japonicum
- S.haematobium

Schistosomiasis (Bilharziasis)
Type
Hypersensitivity
reaction type
Histopathology
Involvement
Shistosomal
antigen-antibody
complex
lateral spine
Hepatic and
intestinal
no spine
involvement
the spine will be
Obstructive
Type III
down
uropathy and
hypersensitivity
bladder cancer
involvement

Antimicrobial treatment is advisable for


-Systemic forms of Salmonellosis ( Typhoid and
Paratyphoid )
-Giardiasis
- Severe Yersinia Infection

Comment / Association
Clinical picture
Gram stain
Laboratory
Treatment of choice

Treatment
Praziquantel is
effective against
all type of
Schistosomiasis

Antimicrobial treatment is ineffective or


unnecessary
- Salmonella gastroenteritis
- Shigella sonnei infection

Mycoplasma pneumonia
Primary atypical pneumonia is caused by Mycoplasma and associated with dry
cough and bullous myringitis.
Both upper and lower respiratory infection
Not seen( too minute)
High titre of IgM cold agglutination
Erythromycin

Disease
Causative organism
Primary herpes infection

Chancroid

Haemophilus ducreyi

Condyloma

Human papilloma virus

Lymphogranuloma
venereum

Sexually transmitted
infection caused by
Chlamydia trachomatis

Chancre

Syphilis is a sexually
transmitted infection
caused by the
spirochete bacterium
Treponema pallidum
subspecies pallidum.

Presentation
-Fever
-Myalgia
-Cervical and inguinal
lymphadenopathy
-Exudative pharyngitis
-Vesicles prior to ulcers
on the oral mucosa, the
labia and cervix. They
are painful.
-Profuse vaginal
discharge
-Neutrophilia
-Doesnt cause systemic
symptoms
-Leads to soft, nonindurated, painful ulcer
-Causes characteristic
large, soft, fleshy,
cauliflower-like
excrescences around
the vulva, urethral
orifices, anus and
perineum.

-Fever
-Arthritis
-Pericarditis
-Painless papule
-Erythema nodosum
-Painful inguinal
lymphadenopathy and
suppurating
-The primary chancre
usually begins as a
single painless papule,
which rapidly becomes
eroded and usually, but
not always, is indurated
with a characteristic
cartilaginous
consistency on
palpation of the edge
and base of the ulcer.

Treatment
-No effective therapy is
available, but acyclovir
is used to reduce
morbidity of the disease
and decrease the
incidence of
recurrences.

-Management consists
of oral erythromycin.

-Frequently used
therapies include
cryosurgery, application
of caustic agents,
electrodesiccation,
surgical excision and
laser ablation.
-Topical podophyllin has
been used with some
success.
-Azithromycin may be of
utility in treatment.

-Penicillin G is the drug


of choice for all stages
of syphilis.

Viral hepatitis

Viral hepatitis
The Most common type of hepatitis over the world is hepatitis-A virus.
Most of hepatitis viruses are RNA except hepatitis-B virus.
The most common cause of chronic hepatitis is hepatitis c-Virus.
Transmission of hepatitis D can occur either via simultaneous infection with hepatitis B (coinfection) or
superimposed on chronic hepatitis B or hepatitis B carrier state (superinfection).
Highest mortality rate in acute infection phase is related to hepatitis-E virus infection.

Hepatitis B virus:

A hepadnavirus is partially double stranded small DNA virus consisting of genomic DNA, core protein
and surface protein.
The core gene encodes for both core antigen and a cleavage product e antigen, which is a good marker
of active viral replication.
Pre S which is closely associated with the surface antigen viral envelope- has specific site for
hepatocytes.
Interferon inhibits viral replication.
Is required for hepatitis D infection.

Hepatitis B markers
HBsAg
Anti-HBs
Anti-HBc

HBeAg
Anti-HBe

Indications
-Indicates acute or chronic infection
-Used for blood bank screening
-Indicates immunity (post-infection or post vaccination)
-Useful because it may be the only serological marker of HBV infection during
the window period in which HbsAg is disappearing, but Anti-HBs is not yet
detectable.
-Doesnt distinguish between acute and chronic infection, and presence
doesnt indicate immunity.
-Indicates high degree of infectivity
-Indicates resolution of acute infection

HCV markers
- Hepatitis C virus (HCV) antibody enzyme
immunoassay
- Recombinant immunoblot assay
- The quantitative HCV RNA polymerase chain reaction

Indications
-Initial assay for detecting hepatitis C virus
-confirmatory test
-Used to measure the amount of virus in the blood to
distinguish active from resolved HCV infection

-Before initiating Tumor necrosis factor inhibitors therapy, Patients should be screened for tuberculosis
and hepatitis B and C, because these drugs have been associated with an increased risk of infections.
- This class of agents includes monoclonal antibodies such as infliximab, adalimumab, certolizumab
pegol, and golimumab.

Treatment of urinary tract infection and pyelonephritis


Uncomplicated lower UTI often respond to Trimethroprim/Sulfamethaxazole or quinolones
during a treatment course of 5-7 days.
Prolonged antibiotic coverage may be required for recurrent or chronic UTIs (e.g.,
Nitrofurantoin).
Acute Pyelonephritis requires broad-spectrum antibiotic coverage such as IV ampicillin and
gentamicin.
Most cases of uncomplicated acute Pyelonephritis can be managed in the outpatient setting.
Fluoroquinolones, such as ciprofloxacin, are the preferred empiric antibiotic treatment for
outpatient treatment of Pyelonephritis.
Urologist referral is necessary for patients who have anatomic obstructions because chronic
reflux can lead to renal failure.

Quick revision
Question
The Best method to prevent plague
syphilis stages

The most important risk factor for developing a


catheter-associated UTI (CAUTI)
The best treatment for subacute and chronic
diarrhea in AIDS patients
EPV is associated with
Protocol for diagnosis and treatment of influenza

Protocol for diagnosis and treatment of dog bites

The most likely type of pharyngitis in a patient


presented with tender anterior cervical
lymphadenopathy and absence of cough

Answer
Eradicate rodent
Chancer (painless) is associated with
primary Syphilis.
Diffuse rashes in palms and palms and
soles and Condylomata lata are associated
with Secondary Syphilis.
Formation of gummata is associated with
Tertiary Syphilis.
Prolonged use of the urinary catheter
Highly active antiretroviral therapy
o Nasogastric carcinoma
o Hodgkin lymphoma (Mixed cellularity)
Influenza should be diagnosed on the basis of
clinical signs and symptoms and should be treated
symptomatically
Dog bites are a common medical problem. Timely
and copious irrigation with normal saline or
Ringers lactate will reduce the rate of infection
markedly. Cultures are usually not helpful unless
the wound appears infected.
Bacterial pharyngitis

Pharmacology
Drug
Amantadine
Oseltamivir (Tami flu)
Imiquimod (aldara)
Trimethroprim/
Sulfamethaxazole
Suppressive therapy with;
Acyclovir
Valacyclovir
Famciclovir
Isoniaziad

Comment
Antiviral and Antiparkinsonism drug that can be used in both adults and
children for up to 8 weeks as a daily prophylaxis for influenza
Antiviral prophylactic drug for influenza, but it is not indicated in children
less than 13 years old
FDA approved to treat external anogenital warts for patients more than
12 years of age
The treatment of choice in Pneumocystis jiroveci which is common with
HIV positive patients
Reduces -but does not eliminate- the risk of transmission of herpes
simplex virus (HSV) to sexual partners

Used as a TB prophylaxis
Laboratory markers and diagnostic methods

Question
The most accurate laboratory test for syphilis
The most useful laboratory test for syphilis
The most appropriate and the most common
staining technique used to identify acid-fast
bacteria (e.g. Tuberculosis)
The most accurate test of herpes encephalitis
The most important initial test on a genital
ulcerative lesion
The most accurate test of genital or skin lesion
The first laboratory marker that becomes abnormal
first after acquiring hepatitis B infection
The best indicatory serological marker of the need
for treatment with antiviral medication in chronic
hepatitis B and the best indicator that pregnant
women will transmit infection to her child
The best correlation with an increased likelihood of
mortality in a patient with hepatitis
Disease
Clinical picture
Laboratory
Most important initial test
Most important serum test
Indication of usage of corticosteroids in
management

Answer
Dark-Field microscopy
Venereal disease research laboratory (VDRL)
Ziehl-Neelsen stain

Polymerase chain reaction (PCR)


Tzank preparation
Viral culture
Surface antigen
E-antigen

Elevated PT

EBV (Infectious mononucleosis)


Malaise, anorexia, fever, sore throat, few palatal petechiea,
cervical lymphadenopathy and splenomegaly.
Lymphocytosis, agranulocytosis, and thrombocytopenia.
Peripheral blood smear shows atypical lymphocytes
Heterophile antibody
Severe hemolytic anemia
Airway obstruction

Notes

Gastroenterology

The most common


1
2
3

Question
Complication of biliary tract infections (Cholangitis and cholecystitis)
Biliary tract infections
Cause of acute cholecystitis

Cause of acute infectious diarrhea

5
6
7
8

Bacterial diarrhea
Cause of infectious diarrhea in day care centers and urban poor areas
Cause of water-borne infectious diarrhea
Cause of diarrhea among hospitalized patients

9
10
11
12
13
14

Upper GI bleeding
Cause of peptic ulcer disease
Cause of lower GI bleeding
Cause of chronic pancreatitis in adult
Cause of chronic pancreatitis in children
Causes of acute pancreatitis

15 Type of pancreatic carcinoma


16 Causative organism in Travelers diarrhea

Answer
Pyogenic liver abscesses
Polymicrobial
Obstruction of the cystic
duct by an impacted
gallstone
Viral (Norovirus, which
include Norwalk virus)
Campylobacter jejeni
Shigella
Giardia lamblia
Pseudomembranous colitis
(Clostridium difficile colitis)
Peptic ulcer disease
Helicobacter pylori
Diverticulosis
Alcoholism
Cystic fibrosis
Gallstones
Alcoholism
Adenocarcinoma
Escherichia coli

Esophageal tumors
The most common type of esophageal cancer worldwide is Squamous-cell carcinoma and it is
mainly attributed to Tobacco and Alcohol.
The most common type of esophageal cancer in USA, Europe and Australia is Adenocarcinoma
and it is mainly attributed to Barretts esophagus.
The most common benign tumor of esophagus is leiomyoma.

Diagnostic and therapeutic tests


Disease

Meconium ileus
Intussusceptions
Ogilive syndrome (paralytic ileus of the colon in old, immobilized people)
Obstructed biliary tract

Test
Gastrografin enema
Air enema
Colonoscopy
ERCP

Laboratory markers and diagnostic methods


Question
Answer
Measurement of both
The best strategy for screening for acute pancreatitis
serum amylase and lipase
Low calcium
The worst prognostic serological marker in Pancreatitis
o WBC count
The most appropriate preliminary diagnostic evaluation for
o Platelet count
inflammatory bowel disease
o Potassium level
o Erythrocyte
sedimentation
rate
Spontaneous bacterial
Diagnostic method for Spontaneous bacterial peritonitis
peritonitis is diagnosed if
the fluid contains
neutrophil at greater than
250 cells per mm fluid
Serum IgA tissue
The most sensitive and specific laboratory test for Celiac
transglutaminase
disease
Antibody (TTG) antibodies
Endoscopy with Small
The Gold standard confirmatory test for Celiac disease
Bowel biopsy
Endoscopy with biopsy
The best diagnostic test for neoplasms of esophagus and
stomach
Prussian blue stain
The most appropriate histological stain for hemochromatosis

Gender
Associated diseases

Labs

Affected ducts
Radiological studies

Primary biliary cirrhosis


Middle-Aged women
Autoimmune disorders (sjogren's
syndrome, CREST, Rheumatoid
arthritis)
-Cholestatic LFTs.
-positive Antimitochondrial
antibodies.
-Elevated cholesterol and HDL.
-Elevated immunoglobulin M.
Intrahepatic only
-Abdominal Ultrasonography,
computed tomography (CT)
scanning, or magnetic resonance
imaging (MRI) are important to
exclude biliary obstruction.
-There is absence of duct
narrowing on ERCP.

Primary sclerosing cholangitis


Younger men
Ulcerative colitis

-Cholestatic LFTs.
- Positive p-ANCA.

Both intra- and extra-hepatic


ERCP and PTC are diagnostic
studies of choice: see multiple
areas of bead-like structuring and
bead-like dilatations of Both
intra- and extra-hepatic.

Definition of Gastrinoma
Epidemiological data

Genetics
The most sensitive and
specific diagnostic test

Disease

Diagnostic clues

Jejunal biopsy

Treatment

Disease
Genetics
Best initial diagnostic test

Other diagnostic test

Treatment

Zollinger-Ellison syndrome
Nonbeta islet cell tumor of the pancreas
In 20 to 60 percent of those with
Zollinger-Ellison syndrome the Gastrinoma is a component of
MEN type 1
An autosomal dominant disorder
Secretin stimulation test

Malabsorption
Celiac disease
Tropical sprue
(Due to bacterial
infection or toxins)
Iron deficiency anemia
Diarrhea and
that doesnt respond to megaloblastic anemia
oral Iron.
several months after
-Dermatitis
travel to tropical
Herpetiformis is
country.
pathognomonic.
-Flat mucosa.
May look very similar to
-Board or absent villi
Celiac disease.
(villous atrophy).
-Inflammation.
Gluten-free diet
-Hydration.
-Sulfonamide or
tetracycline.
-B12 and Folate
supplement.

Alpha-1 antitrypsin deficiency


autosomal codominant disorder
Iron study (Increased serum Iron
and Ferritin, and decreased Iron
binding capacity).
-Most accurate test: Liver biopsy
-Confirmatory tests: Abdominal
MRI and HFE (C282Y) gene
testing.
Phlebotomy

Whipples disease
(Tropheryma whipplei)
-Polyarthritis
-Abnormal skin
pigmentation
-Lymphadenopathy

PAS positive granule in


macrophages.

TrimethoprimSulfamethoxazole
(months to years)

Wilson's disease
autosomal recessive disorder
A slit-lamp examination for
detecting Kayser-Fleischer ring.
-Most accurate test: An
abnormally increased amount of
copper excretion into urine after
taking Penicillamine.
Penicillamine

Hemochromatosis (Bronze diabetes)


Epidemiological data
Tests
-Hereditary Hemochromatosis is the most common -Transferrin saturation and Ferritin tests: Standard
single-gene disease in Western populations.
diagnostic measures for Hemochromatosis.
-Hereditary Hemochromatosis is the most common -Liver biopsy: Formerly, this was the only way to
of several "iron overload" diseases.
confirm a diagnosis of Hemochromatosis, but now
it is less common and could be used to assess the
amount of iron in the liver and can help to diagnose
liver damage (scarring and cancer).
-Magnetic Resonance Imaging: a non-invasive and
accurate alternative to measure liver iron
concentrations
-Genetic analysis for mutation of the HEA gene: can
be used as a diagnostic tool and to screen
asymptomatic relatives.
In persons with symptomatic Hemochromatosis, repeated phlebotomy, by removing excessive
iron stores, results in marked clinical improvement. Specifically, the liver and spleen decrease in
size, liver function improves, cardiac failure is reversed, skin pigmentation (bronzing)
diminishes, and survival rate increases.
If phlebotomy is begun in the precirrhotic stage, liver cancer will not develop.
For unknown reasons, there is no improvement in the Arthropathy or Hypogonadism.

Case
The most important primary management in
gastrointestinal bleeding
Treatment of Pseudomembranous colitis
( Clostridium Difficile colitis)

Most useful prophylactic drug in Travelers


diarrhea

Management
Fluid resuscitation

1-Discontinue offending antibiotic.


2-Metronidazole is the first-line therapy. Oral
administration is best but can be given IV.
3-Vancomycin (given orally) is used for
patients who cannot tolerate metronidazole or
who do not improve.

Bismuth subsalicylate (Pepto-Bismol)


Antibiotics and Antimotiloity drugs are
not used frequently, but
fluoroquinolones are the antibiotics of
choice.

Definitions
Term
Mucosa-associated
lymphoid tissue (MALT)

MalloryWeiss syndrome
Boerhaaves syndrome
Melanosis coli

Meaning
Most MALT lymphoma is related to H.Pylori infection. Studies have shown
that eradication of the infection results in regression of such lymphoma.
This treatment involves a prolonged course of antibiotic therapy aimed at
eliminating the bacteria. MALT lymphoma can progress to diffuse, large, B
cell lymphomas. Combination chemotherapy has been used in treatment of
the diffuse aggressive lymphoma that arises from MALT.
Tear in the mucosa at the junction of the stomach and esophagus
Tear in full thickness (rupture of the esophageal wall)
Brown to black leopard spotting of the colonic mucosa.
A benign condition resulting from abuse of anthraquinone laxatives
such as cascara, senna, or aloe.
The condition resolves with discontinuation of the medication.

Bacterial food poisoning syndromes


Organism
Incubation period
Source of infection (examples)
2-6 hours
Meat and dairy handlers
Staphylococcus aureus
2-8 hours
Reheated fried rice
Brucella cereus
8-14 hours
Reheated meats, poultry and
Clostridium perfringens
legumes
24-48 hours
Foods (e.g., raw eggs, meat,
Salmonella
poultry and seafood)
1-3 days
Food (e.g., raw meat, milk or
Pathogenic Escherichia coli
dairy products and raw fruits and
vegetables) and Contaminated
water.
-Personal contact
1-3 days
Fecally contaminated water
Vibrio cholerae
1-3
days
Fecal-oral spread
Shigella
1-2 days
Canned foods
Clostridium botulinum
During or within 4 weeks of
Antibiotic-associated
Clostridium difficile
discontinuing antibiotic
therapy
2-4 days
Food (e.g., raw or undercooked
Campylobacter
meat, poultry and milk) and
Contaminated water.
-Person to person transmission
via the fecal-oral route
-Contact with infected animals

Syndrome
Familial adenomatous polyposis
Gardner syndrome
Turcot syndrome
Peutz-Jeghers syndrome

FAMILIAL POLYPOSIS SYNDROMES


Symptoms/Signs
>100 polyps in colon and rectum
Polyposis, desmoids tumors, osteomas,
sebaceous cysts
Polyposis, medulloblastoma/glioma
Multiple hamartomatous polyps, melanotic
pigmentation of skin and mucous
membranes

Malignant potential
Yes
Yes
Yes
Small

Quick revision

Question
Abdominal colicky pain with vomiting could
be associated with

Gastroparesis could be associated with

Causes of increase level of amylase, include

The most likely diagnosis of an elderly patient


presented with painless jaundice and
depressed mood

-Definition

-Treatment

Answer
Bowel obstruction
Cholelithiasis
Nephrolithiasis
Longstanding diabetes mellitus
Bowel obstruction
Pancreatitis
Mumps
Pancreatic cancer

Esophageal Variceal Bleeding


Bleeding from esophageal vessels that have increased pressure secondary to
portal hypertension. Variceal bleeding presents as hematemesis or postural
hypotension in a patient with known or suspected portal hypertension.
-Maintain perfusion pressure with normal saline and blood as needed.
-Replace clotting factors with fresh frozen plasma.
-Somatostatin (Octreotide).
-Beta blockers.
-Endoscopic sclerotherapy or banding.
-Balloon tamponade.

Notes

Hematology and Oncology

The most common


Question
1 Presenting symptom of Hodgkin lymphoma
2 Red cell enzymopathy
3 Cause of death in patients with hemophilia A
4 Utilized method for detection of Hemoglobinopathies
5 Cause of spinal cord compression in people with cancer
6 Presenting symptom of multiple myeloma
7 Cause of disseminated intravascular coagulation
8 Form of acute myelogenous leukemia
9 Cause of abnormal bleeding
10 Cause of thrombocytopenia
11 Cause of isolated thrombocytopenia
12 Form of porphyria
13 Inherited bleeding disorder

Answer
Painless lymphadenopathy
G6PD enzyme deficiency
Intracranial hemorrhage
Hemoglobin electrophoresis
Metastasis to the spine
Bone pain
Pregnancy and obstetric complications
t(8;21)
Thrombocytopenia
Increased peripheral destruction
Idiopathic (Immune) thrombocytopenic
purpura
porphyria cutanea tarda
von Willebrands disease

Plasma cell neoplasm


-The most common plasma cell neoplasm is multiple myeloma.
-The most common Immunoglobulin class produced in plasma cell diseases is IgG.
-The monoclonal immunoglobulin in Waldenstrom's macroglobulinemia is of the IgM class. Because IgM
is so large (it circulates as a pentamer), it is restricted to the bloodstream and in high concentrations tend
to cause hyperviscosity of the blood.

Classifications
Metabolic

Neurologic
Cardiovascular
Hematologic

Infectious

Oncologic Emergencies
1. Hypercalcemia (most common)
2. Tumor Lysis Syndrome
3. SIADH (Syndrome of Inappropriate Antidiuretic Syndrome)
1. Spinal Cord Compression
2. Brain metastases/ Increased Intracranial pressure
1. Malignant Pericardial Effusion
2. Superior Vena Cava Syndrome
1. Hyperviscosity due to Dysproteinemia
2. Hyperleukocytosis
3. DIC (disseminated intravascular coagulation)
1. Neutropenic fever
2. Septic shock

Type
Monitor

Antidote

How to monitor anticoagulation therapy?


Unfractionated Heparin
Low molecular weight
heparin
Activated partial
Anti factor Xa
thromboplastin time
Protamine sulphate

Protamine sulphate

Warfarin
Prothrombin time and
International
normalized ratio
Vitamin K

The cause of Thrombocytopenia in the following conditions is correctly assigned in each case:
condition
causes
Acute leukemia
Marrow aplasia; there is hormonal suppression of normal cells by
leukemic cells and mechanical crowding out in the bone marrow.
Systemic lupus erythematosus
Platelet antibodies; may be an initial manifestation.
Gram-negative septicemia
Induces consumption by immune complexes, complement activation
and direct toxemia.
Massive transfusion
Dilution.
Splenomegaly- from any causeSequestration.

-Cisplatin
-Bleomycin
-Vincristine
-Doxorubicin
-Tamoxifen

Inheritance
pathophysiology
Example of Drugs Likely to Induce
Hemolytic Anemia in G6PD
Deficiency

Side effects of chemotherapy


Nephrotoxicity
Pulmonary fibrosis
Neurotoxicity
Cardiotoxicity
Carcinogenic

G6PD enzyme deficiency


X-linked recessive
RBCs are unable to generate reduced glutathione, which protect
hemoglobin from oxidative denaturation.
Antimalarials (e.g., Primaquine, quinine)
Sulfonamides
Nitrofurantoin
Dapsone

Diagnosis and findings of non-hematopoietic diseases in the bone marrow


Disease type
Disease
Bone marrow findings
Bacterial
Tuberculosis
-Aspiration shows acid-fast bacilli
-Biopsy shows epithelioid
granulomas
Unknown
Sarcoidosis
Epithelioid granulomas
(Noncaseating)
Parasitic
Kala-azar
Organism-laden macrophages
(Visceral leishmaniasis)
(Donovan bodies)
Fungal
Aspergillosis
Hyphae with areas of necrosis
Storage disease;
Gauchers disease
Characteristic histiocytic cells
Accumulation of
with a fibrillar inclusion material
glucocerebrosidases
that has an onion-skin pattern
Storage disease;
Niemann-Pick disease
Histiocytic cells with foamy
Accumulation of sphingomyelin
cytoplasm containing lipids and
phospholipids
sea-blue histiocytes
Metabolic
Cystinosis
Histiocytic cells laden with
birefringent cystine crystals
Familial
Osteopetrosis
Compacted intramedullary
(Albers-Schonberg or marble
osseous tissue with very little
bone disease)
hematopoietic tissue
Immunoglobulin
Amyloidosis
Marrow replacement by pale
acidophilic material that stains
positive with Congo red

Megaloblastic Anemia (MCV>100)


-Results from decrease DNA synthesis with normal RNA/Protein synthesis.
-Pathognomonic blood smear
Hypersegmented neutrophil
Type
Vitamin B12 deficiency
Folic acid deficiency
Most common cause
Pernicious anemia
Dietary deficiency
Presentation
Neurologic signs= Peripheral
No Neurologic signs
neuropathy, paraesthesia,
decrease balance and position
sense, worse in legs.
Diagnosis
-Increase serum methylmalonic
-Normal serum methylmalonic
acid level
acid level
-Increase homocysteine levels
-Increase homocysteine levels
Treatment
Vitamin B12 high-dose oral
Oral folic acid supplementation
therapy proven to be equivalent
to parenteral

Quick points

Question
The most likely diagnosis of a middle-aged male
presented with pruritis after showering, plethora of
face and palms and hepatosplenomegaly
The most probable diagnosis of a patient develops a
severe case of hives ten minutes after transfusion
Prolonged PT indicates
Therapeutic range of INR
Biochemical evidence of lead toxicity includes

Proto-oncogenes function

Triad of Plummer Vinson syndrome

Triad of Felty's syndrome

Drug-induced thrombocytopenia

Differences between Hemolytic uremic syndrome and


Thrombotic thrombocytopenic purpura

Before splenoectomy, sickle cell patient should receive


the following vaccines:

Answer
Polycythemia Vera

Selective IgA deficiency


A defect in extrinsic factors
2-3
Elevation of blood lead
An increase in free erythrocyte
protoporphyrin
Control cell growth and
differentiation.
They are transiently
upregulated by growth factors.
They are normal cellular genes
that associated with the
development of a wide variety
of human tumors.
-Iron deficiency anemia
-Esophageal web
-Glossitis
-Rheumatoid arthritis
-Splenomegaly
-Neutropenia
-May be caused by intravenous heparin
-Usually respond to withdrawal of the
drug
-Has a better prognosis than druginduced agranulocytosis
Hemolytic uremic syndrome is similar
to Thrombotic thrombocytopenic
purpura.
But;
HUS occurs in children.
HUS is localized to the kidney.
HUS does not contain
neurologic abnormalities.
H.Influenza vaccine
Meningococcal vaccine
Pneumococcal vaccine

Disease
Age
Presentation

ALL
3-7
-Fever
-Petechiea
-Ecchymoses
-CNS infiltrate
-Pallor

AML
All ages
-Fever
-Petechiea
-Ecchymoses
-Lymphadenopathy
-Splenomegaly
-Pallor

Labs

-Leukocytosis
-Anemia
-Thrombocytopenia
- Lymphoblastosis
- Terminal
deoxynucleotidyl
transferase

-Anemia
-Thrombocytopenia
-Auer rods in
Myeloblasts
-Decreased
Leukocyte alkaline
phosphatase levels

Genetics

An autosomal recessive trait


which results from a single amino
acid substitution of valine for
glutamic acid at the number 6
position in the B chain.

CML
50 years
-Anorexia
-Fever
-Splenomegaly
-Diaphoresis
-Early satiety
-Arthritis
-Bone
tenderness
-Dyspnea
-Dizziness
-Slurred
speech
-Diplopia
-Positive
Philadelphia
chromosome
in myeloid
stem cells ;
t(9;22)
-Decreased
Leukocyte
alkaline
phosphatase
levels

CLL
70 years
-Asymptomatic
then
-Insidious few
symptoms
-Organomegaly
-pallor
-Fatigue
-Lymphadenopathy

Hairy cells
50 years
-Organomegaly

Myelofibrosis
50 years
-Massive
Organomegaly

-Anemia
-Thrombocytopenia
-Lymphocytosis

-Pancytopenia
-Characteristic
hairy cell
morphology in
bone marrow

-Tear drop cells


-Hypercellular
bone marrow
-Increased
Leukocyte
alkaline
phosphatase
levels

Sickle cell disease


What is the definition of acute
chest syndrome?

How to differentiate between


Aplastic crisis and Hemolytic
crisis?

Acute chest syndrome: a vasoocclusive crisis of the pulmonary


Aplastic
Low reticulocyte
vasculature commonly seen in
crisis
count
patients with sickle cell anemia.
Hemolytic High reticulocyte
-Presentation:
crisis
count
Fever
Chest pain
Shortness of breath
Hypoxia
Chest infiltrate
Hemoglobinopathies and conditions causing hemolysis can cause HbA1c measurements to be
falsely low.
In patients with sickle cell trait there is an increased incidence of urinary tract infections.

Hematology pharmacology
Case
Thrombotic thrombocytopenic purpura
Bleeding caused by thrombolytic agents
The most appropriate prophylaxis against DVT (e.g.
after total knee replacement)
The least expensive and best absorbed form of iron
therapy, yielding the most elemental iron absorbed
per gram administered.
The adult dosage is typically 300mg/day and is
gradually increased to 900mg/day if tolerated.

Disease
Histology
Types/Association

Treatment
Emergency large-volume plasma exchange
Cryoprecipitate
Low molecular weight heparin (enoxaprin) and
adjusted dose warfain
Ferrous sulphate

Hodgkin's lymphoma
Presence of Reed Sternberg cell
Lymphocyte depleted
(Worst prognosis)
Lymphocyte rich
(Good prognosis)

Non-Hodgkin lymphoma
Absence of Reed Sternberg cell
Has a bad prognosis and
associated with HIV infection

Mixed
(Associated with EBV infection)
Nodular sclerosing
(Most common)
Type of lymphoma
Hodgkins lymphoma
Non-Hodgkins lymphoma

Types of cells in peripheral smears


Howel -Jolly bodies
Target cell
Heinz bodies and bite cells
Types of anemia
Examples

Microcytic anemia
( MCV<80 fl)
-Iron deficiency
-Thalassemia
-Sideroblastic

Renal complications
-Minimal change glomerulonephritis
-Secondary membranous glomerulonephritis

Possible cause
Sickle cell disease
Thalassemia
G6PD enzyme deficiency
Normocytic anemia
(MCV= 80-100 fl)
-Renal disease

Macrocytic anemia
( MCV>100 fl)
-Folate deficiency
-Vitamin B12 deficiency

Notes

Nephrology

The most common


Question
1
2
3
4
5
6
7

Answer

Form of glomerulonephritis
Cause of glomerular origin hematuria
Cause of acute interstitial nephritis

IgA nephropathy (Bergers disease)


IgA nephropathy (Bergers disease)
Drug-related hypersensitivity reaction

Cause of nephritic syndrome


Cause of end stage renal disease
Cause of anemia of chronic disease
Cause of hypermagnesemia

post-streptococcal glomerulonephritis
Diabetes mellitus
Chronic renal failure
Chronic renal failure

Indications for urgent dialysis


-Uremic complications: pericarditis with effusion, and encephalopathy.
-Pulmonary edema not responding to medical treatment.
-Significant hyperkalemia ( >7 ) with evidence of ECG changes and resistance to medical therapy
-Rapidly rising urea and creatinine.
-worsening severe metabolic acidosis (pH < 7.2 or base excess < -10).
- Intoxication: e.g. methanol and ethylene glycol.

Nephrotic syndrome
-Minimal change disease is the most common cause of Nephrotic syndrome in children.
-Focal segmental glomerulosclerosis is the most common cause of Nephrotic syndrome in adults.
Medical Treatment
Features of Nephrotic syndrome
-Loop Diuretics (e.g. Furosemide)
-Hyponatremia
-ACE Inhibitor (e.g. Enalapril (Vasotec))
-Hypoalbuminemia (Serum albumin<2.5gm/dl)
-Corticosteroids
-Proteinuria > 3.5 gm/ day
-Avoid nephrotoxins (e.g. NSAIDs)
-Hyperlipidemia: is present in vast majority of patients
-Maintain Blood Pressure less than 130/80
with Nephrotic syndrome and typically is characterized
-Control Hyperlipidemia
by a relatively selective increase in LDL. There is
evidence of accelerated atherosclerosis in these
patients.
-Renal vein thrombosis
-Orbital edema
-Thromboembolism
-Infections; due to loss of immunoglobulins in urine
-Coagulability; due to loss of Antithrombin3 in urine

Disorder
Hypocalcemia
Hypercalcemia

Diuretics used to treat Calcium disorders


Drug
Action
Thiazide
Lower urinary calcium and prevent urolithiasis
Furosemide
Inhibit calcium reabsorption

Causes of Hyperkalemia

Renal failure
Drug-induced

-The most common cause of hyperkalemia genuinely is renal failure.

Beta blocker

K+ sparing diuretics ( Spironolactone)

NSAIDs
ACEI, ARBs
Digitalis

Cyclosporine
Heparin
Succinylcholine chloride
(Suxamethonium)

Rhabdomyolysis
Causes
Alcoholism
Hypothyroidism
Drugs

Destruction of RBCs
Potassium supplements
Diabetes mellitus type1
Addisons disease

Sodium imbalance correction complication


An overly rapid increase in serum sodium concentration may produce central pontine
demyelination.
Excessively rapid correction of hypernatremia can lead to cerebral edema.

Disease
ABG
Electrolytes disturbance

Diuretics
Effect

Chronic renal failure


-Metabolic acidosis
-Hypocalcemia
-Hyponatremia
-Hyperkalemia
-Hyperphosphatemia

Thiazide
-Hypokalemia
-Hyperlipidemia
-Hyperuricemia
-Hyponatremia
-Hyperglycemia
-Hypercalcemia

DKA
-Metabolic Acidosis
-Hypokalemia

Loops diuretics
-Hypokalemia
-Hyperlipidemia
-Hyperuricemia
-Hyponatremia
-Hypomagnesaemia
-Hypocalcaemia

Acute pancreatitis
-Metabolic alkalosis
-Hypocalcemia

Spironolactone
-Hyperkalemia
-Hyponatremia

Pharmacology
The most appropriate initial step in a patient presented with hyperkalemia is administration
of IV Calcium gluconate. While administration of IV bicarbonate, (Insulin and dextrose) and
salbutamol are a temporizing measures. Administering oral sodium polystryrease sulfonate
(Kayexalate) is an effective way of permanently removing potassium from the body over a
period of 4-10 hours. Initiating urgent hemodialysis is indicated only if the medical
management of hyperkalemia fails.
The most likely anti-acid responsible for hypophosphatemia is Aluminum Hydroxide.
Gadolinium contrast can cause nephrogenic systemic fibrosis if used in patients with
advanced kidney disease. Contrast has rapid effect on kidney faster than nephrotoxic drugs (which
may appear after 5-10 doses).
The most appropriate initial management of hypercalcemia of malignancy includes fluid replacement
with normal saline to correct the volume depletion that is invariably present and to enhance renal
calcium excretion. The mainstay of treatment for the hypercalcemia of malignancy is intravenous
pamidronate.
Patients with chronic kidney disease (CKD) and those at risk for CKD because of conditions such as
hypertension and diabetes have an increased risk of deterioration in renal function from NSAID use.
Although there appears to be an increased risk of myositis in patients with renal failure treated with
lipid-lowering agents, the risk is relatively low (far below 20 percent). Combination therapy (e.g., an
HMG-CoA reductase inhibitor and a fibric acid derivative) should be used with caution.

Acidosis
Right shift in oxygen-hemoglobin dissociation
curve diminishes affinity of hemoglobin for
oxygen, thus there is increasing in oxygen
delivery to tissues.

Alkalosis
Left shift in oxygen-hemoglobin dissociation
curve increases affinity of hemoglobin for
oxygen, thus there is decreasing in oxygen
delivery to tissues.

Refeeding syndrome: the potentially fatal shifts in fluids and electrolytes that may occur in malnourished patients
receiving artificial refeeding (whether enterally or parenterally).
- The most classic electrolyte abnormality: hypophosphatemia

Syndromes lead to hypokalemic metabolic alkalosis and is chloride resistant (high urinary chloride)
Syndrome
Bartters Syndrome
Gitelmans Syndrome
Liddles Syndrome
Genetics
Autosomal recessive
Autosomal recessive
Autosomal dominant
disorder
disorder
disorder
Resembling
Furosemide (Lasix)
Hydrochlorothiazide
Hyperaldosteronism

Nitrogen accumulation diseases


Renal failure
When the kidneys are unable to excrete urea,
Ammonia may be used for the net synthesis of
nonessential amino acids; thus, the need for
nonessential nitrogen is reduced.

Hepatic failure
Amino acid catabolism is decreased, and so even
normal protein intake may be deleterious.

-Most common cause


-Most common drug-related cause

-Treatment

Acute interstitial nephritis


An adverse reaction to a drug.
Antibiotics, especially Penicillins, Cephalosporins, and sulfonamides.
Corticosteroids may be useful for treating this condition.

Acid-Base balance
The main complication of gastroectomy is metabolic alkalosis.
After prolonged excessive vomiting the patient will suffer from metabolic alkalosis.
The most likely electrolyte imbalance associated with respiratory alkalosis is hypokalemia.
High anion gap metabolic acidosis with calcium oxalate crystals in urine is highly indicative of
ingestion of ethylene glycol.
Aspirin overdose: early respiratory alkalosis, then metabolic acidosis with high anion gap.
Isopropanol poisoning is characterized by ketosis without acidosis.

Primary disorder
Metabolic acidosis
Metabolic alkalosis

Compensation for Metabolic Acid/Base disorders


Expected compensation
Expected PCO2 = (1.5 [HCO3-]) + 8 (range: +/- 2)
Expected pCO2 = 0.7 [HCO3] + 20
(range: +/- 5)

Anion Gap = Na - (Cl + HCO3-)


Delta Gap = Anion Gap - 12 (normal anion gap)

Type of cast

Red cell
WBC
Eosinophil
Hyaline
Broad, Waxy
Granular Muddy-Brown"

Prerenal
Hypoperfusion
secondary to
hypovolemia, blood loss,
dehydration.
Maldistribution, as in
sepsis or liver disease
and low cardiac output.

Association
Glomerulonephritis
Pyelonephritis
Acute (allergic) interstitial nephritis
Dehydration
Chronic renal failure
Acute tubular necrosis
CAUSES OF ACUTE RENAL FAILURE
Renal
Interstitial disease
(interstitial nephritis or
acute tubular necrosis).
Glomerular disease.
Renovascular disease.

Postrenal
Urinary tract obstruction.

Notes

Endocrinology

The most common


Question
1 Cause of Charcot foot
2 Form of diabetic neuropathy
3 Cause of hypoglycemia in diabetic patients
4 Cause of death in diabetic patients
5 Cause of hypothyroidism
6 Cause of hyperthyroidism
7 Etiologies of hypercalcemia
8 Presentation of primary hyperparathyroidism
9 Cause of primary hyperparathyroidism
10 Cause of secondary hyperparathyroidism
11 Cause of hypoparathyroidism
12 Reproductive disorders in men

Answer
Diabetes mellitus
Distal symmetric polyneuropathy
Insulin overdose
Coronary heart disease
Hashimoto's thyroiditis
Graves disease
Primary hyperparathyroidism and malignancy
Asymptomatic hypercalcemia
Parathyroid adenoma
Chronic renal failure
Surgical removal of the parathyroid glands
during a neck procedure
Gynecomastia
Hypogonadism

Laboratory marker
Question
The most accurate measure of severity of DKA
First initial sign to detect diabetic nephropathy
The major form of thyroid hormone in the blood which has a
longer half-life
The most active (Potent) thyroid hormone
The most appropriate laboratory tests to confirm congenital
hypothyroidism
The most initial appropriate test for menstrual irregularities
The best test used to monitor response to thyroxin replacement
therapy
Subacute thyroiditis is associated with
The most appropriate test to screen for late-onset male
hypogonadism
The most appropriate initial test to rule out adrenal insufficiency

Disease
Laboratory diagnostic
tests

Cushings syndrome
Initial screening test(he most
specific diagnostic test): 24hour urinary free cortisol
level
Another excellent screening
test(Confirmatory test): Low
dose Dexamethasone
suppression test

Answer
Serum bicarbonate
Microalbuminurea
T4
T3
T4 and TSH
TSH
TSH
High ESR
Total testosterone
Morning serum cortisol
Acromegaly
Screening test/The most
sensitive laboratory test: IGF1
(Somatomedin C)
Confirmatory test/the most
specific laboratory test: Oral
glucose suppression test.

Presentation
Hypertension with
Hypokalemia

Hypotension with
Hyperkalemia

Disease
Primary
Aldosteronism
(Conns syndrome)
Secondary
Aldosteronism
Cushing's Disease
Adrenal
insufficiency
(Addison disease)

part
The adrenal cortex:
the outer part of the
gland and comes from
mesoderm
The adrenal medulla:
the inner part of the
gland and comes from
neural crest

Aldosterone
High

Cortisol
Normal

Renin
Low

High

Normal

High

Low
Low

High
Low

Low
High

Adrenal gland
Primary regulatory
Anatomy
control
Renin-Angiotensin
Zona glomerulosa
ACTH, hypothalamic
Zona fasciculata
CRH
ACTH, hypothalamic
Zona reticularis
CRH
Preganglionic
Medulla
sympathetic fibers
(Chromaffin cells)

Diabetic ketoacidosis (DKA) is


best defined as:

Secretory products
Aldosterone
Cortisol, Sex hormone
Sex hormone
(e.g., Androgen)
Catecholamines
(Epinephrine, and
Norepinephrine)

Serum PH<7.3
Blood glucose of more than 11mmol/L
Positive serum acetone

-The most accurate measure of the severity of DKA is serum Bicarbonate. If the Bicarbonate level is low,
the anion gap is increased.
Multiple Endocrine Neoplasia syndromes
-Autosomal dominant inheritance with incomplete penetrance.
-Type 1 (Wermers syndrome)
Familial Primary Hyperparathyroidism
Pituitary Adenoma
Pancreatic islet tumors
-Type 2a (Sipples syndrome)
Medullary thyroid carcinoma
Pheochromocytoma
Hyperparathyroidism
-Type 2b
Mucosal neuromas
Medullary thyroid carcinoma
Marfanoid body habitus
Pheochromocytoma

Diabetes Insipidus
(Lack of Antidiuretic hormone)
Hypernatremia
Dehydration

Most common cause worldwide


Most common cause of in the western
world
Presentation

Laboratory tests

Disease
Clinical presentation

The syndrome of inappropriate antidiuretic


hormone secretion (SIADH)
Hyponatremia
Normal volume status (euvolemic)
Addisons disease
Tuberculosis
Autoimmune disease

weight loss
Hyperpigmentation
Hypotension
Fatigue
Loss of appetite
Depression
High ACTH
Low cortisol
Low Na
High K
High Ca

Common autoimmune thyroid diseases


Graves disease
Hashimoto disease
(Diffuse toxic goiter)
An autoimmune cause of
An autoimmune cause of
hyperthyroidism, with heat
(usually) hypothyroidism, with
intolerance, palpitation,
cold intolerance, rubbery goiter,
tachycardia, tremor and
facial puffiness, and sparse,
opthalmopathy.
coarse hair.

Endocrinology pharmacology
Metformin is the only hypoglycemic agent shown to reduce mortality rates in patients with
type 2 diabetes mellitus.
Type of acidosis after usage of metformin is lactic acidosis, which is uncommon side effect.
Metformin is contraindicated in patients with renal insufficiency.
Sulfonylurea initially acts by increase secretion of insulin by beta cells.
Pioglitazone can cause fluid retention and therefore would not be a good choice for a patient
with cardiomyopathy.
In a patient receiving Levothyroxine, a low TSH level usually indicates over replacement.
Exogenous testosterone leads to Infertility.
Abrupt cessation of exogenous glucocorticoids leads to adrenal insufficiency.

Quick revision
Diagnostic criteria for Diabetes Mellitus

Risk factors that increase diabetic


nephropathy

Hemoglobin A1c level 6.5%


Fasting plasma glucose level 126mg/dL
Random glucose level 200 mg/dL in a
patient with symptoms of diabetes
2-hour oral glucose tolerance test value
200 mg/dL

Poor glucose control

Long standing diabetes mellitus


Hypertension
Hypercholesterolemia
Renal disease
Pregnancy

Metformin is well tolerated and there is


good data to show it helps prevent type 2
diabetes mellitus in high-risk patients.
Pioglitazone has been shown to slow the
progression from Prediabetes to diabetes,
but it has more side effects and is more
expensive than metformin.

Pharmacologic Treatment
recommendations for Prediabetes

Diabetes mellitus complications

The following drugs have been associated


with impairment of glucose tolerance
Why diabetic patients should avoid beta
blockers?

Macrovascular
complication

Microvascular
complication

Brain, heart and


peripheral
vasculature
Retina, kidneys and
nerves

Bendrofluazide (Thiazide diuretics)


Estrogen
Prednisolone (corticosteroids)
Because they could mask hypoglycemic features.

Drug class
Biguanide

Insulin secretagogue

Insulin sensitizers
(Thiazolidinedione)

Alpha-glucosidase inhibitors

Dipeptidyl peptidase-4
inhibitors

Condition
Primary
hypothyroid
Secondary
hypothyroid
Primary
hyperthyroid

Diabetes Medications
Mechanism of action
-Sensitizes peripheral tissues to
insulin and increases glucose
uptake.
-Decreases hepatic glucose
production
-Stimulates insulin release from
beta cells by causing K channel
closure leads to depolarization
and Ca mediated insulin
release.

-Sensitizes peripheral tissues to


insulin and increases glucose
uptake.
-Decreases free fatty acid
release from adipose tissue.
-Bind to nuclear receptor.
-Decreases carbohydrates
absorption from GI by inhibiting
brush border alpha-glucosidase.
-Inhibits degradation of
endogenous antihyperglycemic
incretin hormones.
-Incretin hormones stimulate
insulin secretion, inhibit
glucagon release and delay
gastric emptying.

Drug name
Metformin

Sulfonylureas:
Glyburide
Gliclazide
Glimepiride
Non-Sulfonylureas:
Repaglinide
Nateglinide
Rosiglitazone
Pioglitazone

Acarbose

Sitagliptin (Januvia)

Laboratory findings in thyroid dysfunction


T4
RTU(T3 UPTAKE)
FREE T4 Index
Decreased
Decreased
Decreased

TSH
Increased

Decreased

Decreased

Decreased

Decreased

Increased

Increased

Increased

Decreased

Drug class
Antithyroid
agent

Drug name
Propylthiouracil

Thyroid
hormone

Indications
Hyperthyroidism

Methimazole

Levothyroxine

Hypothyroidism

Contraindications
-Hypersensitivity
-Renal and liver
failure

Side effects
-Nausea, vomiting
-Rash
-Drug-induced
hepatitis
-Agranulocytosis

-Pregnancy
-Lactation

Drug-induced
hepatitis
Symptoms of
hyperthyroidism

Notes

Rheumatology

Quick revision
Question

Answer

The most common joint involved in Pseudgout


The most common cause of acute monoarthritis in elderly
The first line treatment of Rheumatoid Arthritis according to
latest guidelines
Advantages of omega-3 and omega-6 fatty acids in patients with
rheumatoid arthritis

The most typical characteristic for polymyalgia rheumatica

Knee
Acute synovitis (Pseudgout)
Methotrexate
Substitution of omega-3 fatty acids found in
certain fish oils or dietary omega-6 essential
fatty acids has been shown to provide
symptomatic improvement in patients with
rheumatoid arthritis, but has not been shown to
alter the disease process
A dramatic response to corticosteroids

Pencil-in-cup appearance on x-ray is associated with


Systemic lupus erythematosus malar rash description

Psoriatic arthritis

Heart condition that is commonly associated with


connective tissue diseases (.e.g. Rheumatoid arthritis, SLE,
Mixed connective tissue disease (sharps syndrome), Stills
disease)

Pericarditis

The strongest marker to exclude discoid lupus from systemic


lupus erythematosus
In an acute lupus flare

Presence of Anti-double-stranded DNA

In Systemic lupus erythematosus, the first


criterion is a malar rash characterized by an
Erythema over the cheeks and nasal bridge (but
sparing the nasolabial folds, which is in contrast
to the rash of dermatomyositis). It lasts from
days to weeks and is occasionally painful or
pruritic.

The most common cause of secondary Raynauds phenomenon


Amyloidosis definition

Osteoarthritis
Bouchards nodes: Bony swelling at the proximal
interphalangeal joints of the fingers.
Heberdens nodes: Bony swellings at the distal
interphalangeal joints of the fingers.

Complement level drops


Anti-double stranded DNA levels rise
Scleroderma
Long-standing rheumatoid arthritis is currently
one of the most common causes of systemic
amyloidosis. This may give rise to a complex
clinical picture resulting from amyloid deposition
in the skin, kidneys, tongue gastrointestinal tract
and peripheral nerves.
Patient may present with carpal tunnel
syndrome, skin plaques in the axillary region,
nephrotic syndrome, hepatosplenomegaly,
macroglossia and chronic diarrhea with
malabsorption.

Rheumatoid Arthritis
Boutonniere deformity: Characterized by fixed flexion of
proximal interphalangeal joints and extension of the
distal interphalangeal joints.
Swan neck deformity: Characterized by hyperextension
of the proximal interphalangeal joints and fixed flexion
of the distal interphalangeal joints.

Disease
Classic age/Sex
Location
ESR
EMG/biopsy
Findings

Treatment

Fibromyalgia
Young adult women
Various

Polymyositis
Female aged 40-60 years
Proximal muscles

Normal

Elevated

Normal
-Anxiety
-Stress
-Insomnia
-Point tenderness over
affected muscles
-Antidepressants
-NSAIDs
-Pregabalin
-Rest

Abnormal
-Elevated CPK
-Higher risk of cancer

Polymyalgia Rheumatica
Female> age 50 years
Pectoral and pelvic girdles,
and neck
Markedly elevated
(often>100)
Normal
-Associated with temporal
arteritis

Steroids

Steroids

Inflammatory Myopathies
Characterized by:
-Slowly progressive, symmetric proximal muscle weakness.
-Elevated muscle enzyme (Creatine Kinase and aldolase).
-Biopsy: Lymphocytic inflammation.
-20% are associated with other autoimmune diseases.
-10% are associated with malignancies.
Disease
Dermatomyositis
Characteristics
-Myopathy
-Dermatologic manifestation
(Heliotrope rashes, Gottrons
papules, Shawl (or V-) sign,
Periungual telangiectasia,...)
-Calcinosis
Treatment
-Prednisolone
-Methotrexate
-Intravenous immunoglobulin
-Azathioprine
-Cyclophosphamide
-Rituximab

Polymyositis
-Myopathy
-No rash
-No calcinosis

Drug induced lupus

Description
Often mild
Reversible after drug cessation
ANA may persist for years

Common drugs that may induce lupus


Hydralazine
Procainamide
Quinidine
Isoniazid
Chlorpromazine
Methyldopa

Disease

Gout (Monosodium urate)

History
Physical Findings

Male, binge drinking, acute onset


afterward.
The first big toe is affected.

Crystal shape
Crystal Birefringence

Needle shaped
Negative

Pseudgout (Calcium
pyrophosphate
deposition disease)
Hemochromatosis or
Hyperparathyroidism.
The wrists and knees are
affected.
x-ray features: Chondrocalcinosis
Rhomboid
Positive

Notes

Neuroscience

The most common


1
2
3
4
5
6
7
8
9
10
11
12
13
14
15
16

Question
Presentation in multiple sclerosis
Complication of seventh cranial nerve palsy
Neurological deficit of untreated bacterial meningitis
Cause of aphasia
Cause of non-traumatic subarachnoid hemorrhage
Causative organism of encephalitis
Toxin associated with seizures
Initial manifestation of increasing intracranial pressure
in victim of head trauma
Cause of intracerebral hemorrhage
Sites for hypertensive hemorrhage
Headache in adults
Forms of syncope
Form of diabetic neuropathy
Cause of seizure in elderly
Pattern of multiple sclerosis
Cause of death after severe head injury

Answer
Blurred vision
Corneal ulceration
Eight cranial nerve deficit or deafness
Cerebrovascular disease
Rupture of a cerebral aneurysm
Herpes
Alcohol
Change of level of consciousness
Hypertension
Caudate and Putamen
Tension headaches
Neurally mediated or Vasovagal types
Distal symmetric polyneuropathy
Ischemia-stroke, transient ischemic attack
Relapsing-remitting category
Elevated intracranial pressure

Diagnostic or radiological tool

Question
The most accurate test (the most sensitive and
specific investigation) for Multiple sclerosis
The most accurate test for Herpes encephalitis
The most well-validated tests for confirming brain
death

Answer
MRI
PCR
An electroencephalogram (EEG)

Neuroscience pharmacology
The most important step in management of a Parkinson disease patient presented with psychosis
is antidopaminergic (e.g., Clozapine: antipsychotic drug with fewest extra-pyramidal effects).
The best initial therapy for bacterial meningitis is Ceftriaxone and vancomycin.
Initial therapy of cluster headache includes inhalation of 100% oxygen for 15 minutes.
Agent of choice for treatment of trigeminal neuralgia is Carbamazepine.
Agent of choice for treatment of giant cell arteritis is Corticosteroids.
Aspirin, Acetaminophen, Ergotamine and caffeine combination, Sumatriptan, Amitriptyline,
Calcium channel blockers and Beta-blockers are antimigrainous agents that have been used for
treatment and prophylaxis for different types of headache (Tension, Migraine and Cluster).

Quick revision
o

Question
Brown-Squard
syndrome definition

Features of Pons lesion

Answer
Brown-Squard syndrome is an incomplete spinal cord lesion
characterized by Ipsilateral spastic paralysis below the level of the
lesion and Abnormal reflexes (lateral corticospinal tracts), Ipsilateral
loss of tactile discrimination, vibratory, and position sensation below
the level of the lesion (Posterior (dorsal) column-medial lemniscus
pathway) and Contralateral loss of crude touch, pain and temperature
sensation usually occurs 2-3 segments below the level of the lesion (
lateral spinothalamic tracts).
A lesion within Pons cause ipsilateral facial droop with contralateral
hemiparesis.
Resting tremor

Type of tremor in
Parkinson's disease
o The most dangerous
Dysautonomia
complication of
Involvement of the respiratory muscle
o Guillain-Barre
syndrome
o The most probable
Atherosclerosis
pathophysiological
mechanism for stroke in
a patient with Diabetes
mellitus and
Hypertension
o Indicators of basal skull Battle's sign - is ecchymosis of the mastoid process of the
fracture
temporal bone
Raccoon eyes - is periorbital ecchymosis i.e. "black eyes"
CSF rhinorrhea
Petrous fracture- is bleeding from the external auditory meatus or
CSF otorrhea
Subconjunctival hemorrhage
Hallmark of Amyotrophic
The presence of upper and lower motor neuron signs
Lateral Sclerosis
Transient ischemic attack
Reversible neurologic deficit that lasts from a few minutes to no more
definition
than24 hours.

Signs of Meningeal Irritation


Nuchal Rigidity
Involuntary muscle
spasm limits passive
neck flexion.

Spinal Rigidity
-Erector spinae muscle
spasm limits spine
movement.
-Opisthotonos (rigid
arched back) may occur.

Kernig's Signs
Hamstring muscle pain
when examiner lifts the
supine patients
extended leg.

Brudzinski's Sign
Involuntary hip and
knees flexion due to
passive neck flexion.

parkinsonism
Causes
Drug-induced

Toxins

Viral encephalitis
Neural damage

Treatment

-Reserpine
-Tetrabenazine
-Phenothiazine
-Methyl-PhenylTetrahydropyridine
-Manganese
-Carbon monoxide
Japanese B
-Supra nuclear palsy
-Shy Drager syndrome
-Cerebral tumor
-Wilson's disease
-Huntington's disease
-Neurosyphilis

Co-careldopa
( Contains Ldopa )
Benztropine

The first medication used to


increase dopaminergic activity in
the basal ganglia
Used for anticholinergic side
effects
Selegiline
An MAO inhibitor
( potentiates dopamine )
Amantadine
An antiviral drug ( also
potentiates dopamine ) used as
a second line drug
Apomorphine Used for on-off fluctuations
Benzhexol
Indicated when there is mild
parkinsonism or when there is
oculogyric crisis
Ropinirole
A non-ergot dopamine agonist
that binds the dopamine
receptors D3 and D2

Locked-in syndrome
Definition
-A condition in which a patient is aware but cannot
move or communicate verbally due to complete
paralysis of nearly all voluntary muscles in the body
except for the eyes.

Wernickes encephalopathy
Presentation

Consider it in
Treatment
Complications

Possible causes
-Amyotrophic lateral sclerosis
-Medication overdose
-Multiple sclerosis
-Damage to nerve cells, particularly destruction of
the myelin sheath, caused by diseases (e.g. central
pontine myelinolysis)
-A stroke or brain hemorrhage, usually of the
basilar artery
-Traumatic brain injury

Thiamine (vitamin b1) deficiency.


Triad of:
1- Opthalmoplegia ,Nystagmus
2- Ataxia (wide based gait)
3 -Confusion
Alcoholics
IV thiamine
korsakoff's psychosis , death

Eye Opening

Best Verbal Response

Best Motor Response

Glasgow Coma Scale


Spontaneous: 4
To speech: 3
To pain: 2
No Response: 1
Oriented: 5
Confused: 4
Inappropriate words: 3
Incomprehensible sounds: 2
No Response: 1
Obeys: 6
Localizes : 5
Withdraws to pain: 4
Abnormal Flexion (Decorticate Posturing): 3
Abnormal Extension (Decerebrate Posturing): 2
No Response: 1

Idiopathic intracranial hypertension (IIH)= Benign intracranial hypertension (BIH)= Pseudotumor cerebri
(PTC)
Definition
A neurological disorder that is characterized by increased
intracranial pressure in the absence of a tumor or other diseases.
Presentation
-Headache (Most common symptom)
-Nausea and vomiting
-pulsatile tinnitus
-Double vision
- swelling of the optic disc in the eye (If there is no treatment)
Most commonly occur in
Women aged 20-40, especially those with obesity.
Most common cranial nerve palsy
6th
Complication
Vision loss
Treatment
Some respond to medication (with the drug acetazolamide), but
others require surgery to relieve the pressure.

Causative organism
Most presenting feature
Cause of seizures
Diagnostic tool
Confirmatory diagnostic method
Treatment

Neurocysticercosis
Taenia solium ( Pork tapeworm )
Focal seizures
Localized inflammation that accompanies their degeneration in the
cerebral cortex when calcified cysts occur
CT or MRI may show granulomatous cysts
An enzyme-linked immunotransfer blot
Albendazole

Type of headache
Classic migraine

Common migraine

Cluster

Tension

Depression headache
Trigeminal neuralgia

Giant cell arteritis


(Temporal arteritis)

Clinical features
-Associated with a prodromal aura, which is usually a transient visual, motor, or sensory
phenomenon.
-Headache is typically unilateral and pulsating.
-Headache is preceded by a prodrome.
-Headache may persist for 1-2 days.
-Pain may vary from mild to severe.
-Some prodromal symptoms may be severe and produce transient hemiplegia, aphasia, or
hemisensory deficits.
-Not associated with a prodromal aura.
-Pain is unilateral or bilateral and is usually intense.
-Pain usually affects the eyes, frontal regions, and temples.
-Headache typically lasts for a day or longer.
-Both types of migraines tend to begin in adolescence or early adulthood and may be
associated with vomiting.
-Recur over periods of weeks to months, followed by periods with no headache.
-Cluster headaches are most common in middle-aged men with leonine facies and a
history of heavy smoking or drinking.
-Each headache typically lasts 30 minutes to 2 hours.
-Pain is unilateral and occurs around the eyes.
-Horner syndrome may present.
-Headache may be associated with nasal congestion/rhinorrhea.
-Pain radiates to the ipsilateral neck or jaw.
-Ipsilateral conjunctival injection and ipsilateral facial redness may be seen.
-Alcohol is a common trigger.
-Characterized by a feeling of tightness (band-like), pressure and constriction.
-These headaches are often associated with stressors.
-Pain is commonly suboccipital and nonthrobbing.
-Pain slowly increases and may last for many hours or even days.
-Associated with prolonged positioning of the head and neck.
-Often worse in the morning.
-Frequently associated with other manifestations of depression.
-Appears in the middle to later part of life and is more common in women.
-Characterized by momentary, sudden, lancinating facial pain.
-Pain typically arises on one side of the mouth and then radiates to the eye, ear, and/or
nostril of the ipsilateral side.
-Pain may be precipitated by touch, movement, breezes, or eating.
-Attacks tend to increase in frequency.
-Neurologic examination is usually normal unless an underlying disease is present (e.g.,
multiple sclerosis).
-Granulomatous medium-and large-artery vasculitis that affects persons older than 50
years of age.
-The disease may coexist with polymyalgia rheumatica (characterized by aching
extremities, stiffness, fatigue, and headache).
-Symptoms include those of claudication (pain, pallor, paresthesias, and pulselessness),
headache, visual changes and scalp tenderness.
-Common sites of claudication include jaw (indicating temporal arteritis), tongue and
extremities.
-Visual changes include blurring, ptosis, diplopia and partial or complete blindness (due to
ophthalmic vessel arteritis).
-New onset of headache in an elderly person with fever and anemia suggests the
diagnosis.
-Tenderness over the affected portion of the temporal artery may be elicited, and this
area should undergo biopsy immediately because of the risk of blindness.

Type of motor neuron lesion


Muscle bulk (Wasting)
Muscle tone
Muscle power
Deep tendon reflexes
Superficial reflexes
Plantar response
Fasciculation

UMN
No
Increases
Paralysis affects movements of
group of muscle
Exaggerated
Clonus may be present
Lost
An extensor plantar response
(upgoing)
Absent

LMN
Yes
Decreases
Individual muscle is paralyzed
Diminished
No clonus
Not affected
Flexor response
(downgoing)
Present

Normal pressure hydrocephalus


Mild impairment of memory typically develops gradually over weeks or months, accompanied by
mental and physical slowness.
The condition progress insidiously to severe dementia.
Patient develops an unsteady gait and urinary incontinence, but there are no signs of increased
intracranial pressure.

Most common type of pituitary tumor


Classical physical finding
Medical treatment

Pituitary tumor
Prolactinoma
bitemporal hemianopia
Dopamine agonist (e.g. bromocriptine)
Sodium correction

Condition
Chronic hypernatremia

Chronic hyponatremia

Comment
Correction of chronic hypernatremia should be accomplished
gradually over 48-72 hours to prevent neurologic damage
secondary to cerebral edema.
Correction of chronic hyponatremia should be accomplished
slowly >27 hours to prevent central pontine myelinosis
(Paraparesis/quadriparesis, dysarthria and coma).

Guillain-Barr syndrome
-Manifests with ascending paralysis (first the lower,
then the upper extremities are involved) and
results from a chronic inflammatory response
leading to demyelination of peripheral nerves. It is
often preceded by an upper respiratory tract
infection.

Myasthenia gravis
-Characterized by fluctuating muscle weakness that
usually begins in the ocular muscles, resulting in
diplopia and ptosis. The disorder is due to impaired
cholinergic transmission at the neuromuscular
junction.

In myasthenia gravis
Antibodies to acetylcholine receptors are present.
Antibodies to striated muscle are associated with the presence of a thymoma.
Tendon and pupillary reflexes are normal.
Large doses of anticholinesterases may precipitate a cholinergic crisis (presence of cholinergic
features of sweating, salivation, diarrhea and miosis.

The most common primary central nervous system tumors


The most common CNS tumor
Metastatic
The most common CNS primary tumor in adults
Glioblastoma
The most common CNS primary tumor in pediatrics
Medulloblastoma
Acute head injury
Acute epidural hematoma produces a biconvex,
Acute subdural hematoma produces a crescent
lens-shaped collection
shaped hematoma
-Initial management includes hyperventilation, diuretics and fluid restriction.
-If there is a midline shift or anisocoria (inequality of pupils), patient must undergo surgical evacuation.
Type of aphasia
Wernicke's aphasia
Broca's aphasia
Conduction aphasia
Global aphasia

Association
Receptive, fluent aphasia. The lesion is posterior to the central sulcus.
Expressive, nonfluent aphasia. The lesion is anterior to the central sulcus.
Disturbance in repetition. Pathology involves the connections between
Wernike's and Broca's areas.
Disturbance in all areas of language function. Often associated with a
right hemiparesis.

Notes

Pediatrics

The most common


1
2
3
4
5
6
7
8
9
10
11
12
13
14
15
16
17
18
19
20
21
22
23
24
25
26
27

Question
Congenital heart disease
Acynotic heart defect
Cause of cyanotic heart defect
Cause of cyanotic heart defect in neonates
Pathologic arrhythmia in pediatric age group
Cause of recurrent abdominal pain in children 416 years of age
Organic cause for recurrent abdominal pain in children
Cause of urinary tract infection in infant
Vertically transmitted infection (TORCH)
Pathogen in occult bacteremia in children under 3 years of age
Organism causing otitis media in children
Causative organism in gastroenteritis
Bacterial organism in gastroenteritis
Infectious cause of pediatric vaginal discharge
Cause of seizures in neonate
Cause of convulsions in young children
Neuromuscular disorder of childhood
Cause of death in Duchene muscular dystrophy
Cause of arrest in children
Reason of admitting a newborn to a level 2 or 3 NICU
Cause of bronchoectasois
Cause of neonatal mortality
Cause of death during the first 12 months of life
Morbidity seen among patients with Juvenile rheumatoid arthritis
Type of elevated bilirubin
Cause of rectal bleeding
Cause of infant botulism

28
29
30
31
32
33
34
35
36
37
38

Malformation of the head and neck


Type of karyotype abnormality associated with infertility
Etiology of purpura
Form of vasculitis
Cause of readmission of the infants
Organism involved in lung abscess
Complication of lung abscess
Cause of endobronchial obstruction
Lysosomal storage disease
Form of spinal muscular atrophy
Congenital cause of hydrocephalus

Answer
Ventricular septal defect
Ventricular septal defect
Tetralogy of Fallot
Transposion of great vessels
Supraventricular tachycardia
Functional abdominal pain
Urinary tract infection
Escherichia coli
Cytomegalovirus
Streptococcus pneumoniae
Streptococcus pneumoniae
Rotavirus
Campylobacter jejeni
Group A streptococcus
Hypoxic-Ischemic encephalopathy
Febrile convulsions
Duchenne muscular dystrophy
Respiratory failure
Respiratory
Respiratory distress
Cystic fibrosis
Prematurity
Sudden infant death syndrome
Uveitis
Unconjugated bilirubin
Milk protein colitis
Ingestion of Clostridium botulinum
spores in honey
Unilateral cleft lip
Klinefelters Syndrome
Trauma
Henoch-Schonlein purpura
Jaundice
Staphylococcus aureus
Intracavitary hemorrhage
Foreign body aspiration
Gaucher disease
Werdnig-Hoffman disease (SMA type 1)
Neural tube defects

Mental retardation
Most common cause
Second most common cause

Down's syndrome
Fragile-X syndrome

Short stature

Familial
Constitutional

Most common tumors

Most common tumor


Most common childhood acute lymphoblastic leukemia, which
has a good prognosis and can be treated with mild therapies
Most common soft tissue tumor
Most common intra-abdominal tumor
Most common renal tumor
Most common Benign laryngeal tumor
Most common primary intraocular malignancy
Most common Optic nerve tumor
Most common Optic chiasm tumor

Acute lymphoblastic leukemia


Immature B cell type
Rhabdomyosarcoma
Neuroblastoma
Wilm's tumor
Recurrent respiratory papillomatosis
Retinoblastoma
Optic glioma
Craniopharyngioma

Respiratory

The most common cause of croup (steeple sign) is parainfluenza virus type 1 and 2, the RSV.
The most common cause of epiglottitis (positive thumbprint sign) is HIB.
Whooping cough is caused by Bordetella pertussis (Butterfly pattern).
The most common cause of Bronchiolitis is Respiratory syncytial virus.
The most common cause of pneumonia overall is Streptococcus pneumoniae.
The most common cause of acute infectious pharyngitis is streptococcus pyogenes.

General Characteristics
-Autosomal recessive
disease
-The most common type
(90%) is due to 21hydroxylase deficiency.

Congenital adrenal hyperplasia


Labs
Diagnosis
-Serum electrolytes:
increased
-Hyponatremia
17-Hydroxyprogesterone
-Hypochloremia
-Hyperkalemia
-Hypoglycemia
-Adrenal labs:
-Low cortisol
-Low Aldosterone
-High renin

-Surgically:
-Early correction of female
genital abnormality.

-Sex hormone
abnormalities depend on
enzyme deficiency:
-Virilizing if 21-Hydroxylase
or 11-Hydroxylase
-Testosterone increased in
girls
-Androstenedione
increased in girls and boys

Causes of Conjunctivitis in neonates


At first day of life
2-7 days
More than 7 days
More than 3 weeks

Treatment
-Medically:
-Rehydration
-Correct Hypoglycemia
-Cortisol and
Mineralcorticoids

Chemical
Neisseria gonorrhea
Chlamydia
Herpes

Vaccination
Vaccine type
Live, attenuated

Inactivated/Killed

Toxoid (inactivated toxin)


Subunit/conjugate

Examples

MMR
Chicken pox vaccine
Oral polio vaccine
Rotavirus vaccine
BCG vaccine
Influenza (nasal spray)

Injectable Polio vaccine


Hepatitis A
Rabies
Diphtheria, tetanus (part of DTaP combined immunization)

Hepatitis B
Influenza (injection)
Haemophilus influenza type b (Hib)
Pertussis (part of DTaP combined immunization)
Pneumococcal
Meningococcal
Human papillomavirus (HPV)

Oral Polio vaccine

Injectable Polio vaccine

Live-attenuated
Lifelong immunity
Local gut and systemic immunity
May cause paralytic disease

Inactivated
Required booster dose every 4-5 years
Minimal gut immunity
No risk of paralytic disease

varicella vaccine
-Two doses of varicella vaccine are recommended for all children unless they are immunocompromised, in which
case they should not be immunized against varicella, or with other live-virus vaccines.
- Shingles is evidence of prior varicella infection and is a reason not to vaccinate with varicella vaccine.

Hepatitis b vaccine

Vaccination of baby against hepatitis B must be at birth, 2 month and 6 month. If the mother had
hepatitis B +ve, the child should be vaccinated and given immunoglobulin in the first 24 hours. If the
status of mother is unknown, we must vaccinate the child and ask for mother HbsAg.

Timing
Question
Anterior fontanel closes between
Posterior fontanels close by
Food introduction for infants should be at

Answer
9-18 months
The end of second month
4-6 months of age

Retinoblastoma
-Retinoblastoma is the most common primary intraocular malignancy of children.
-The mode of inheritance of hereditary form of retinoblastoma is Autosomal dominant.
-Familial cases of retinoblastoma have predisposition to second malignancies, particularly osteosarcoma of femur.
-A small percentage of retinoblastoma cases are caused by deletions in the region of chromosome 13 (13q14)
containing the RB1 gene.

First line
Second line
Third line
Fourth line

Management of status epilepticus


Benzodiazepines (lorazepam, and Diazepam)
Phenytoin/ Fosphenytoin
Phenobarbital
Neuromuscular blocking agent (Succinylcholine, Vecuronium or pancuronium) for intubation
and general anaesthesia (Midazolam or Propofol).

Disease
Osteogenesis imperfecta
Ehlers-danlos syndrome
Marfans syndrome
Mucopolysaccharidosis
Achondroplasia

Component
Protein
Fat
Carbohydrates
Albumin-Iron-Immunoglobulin
Casein
Whey
Calories

Abnormal gene product


Procollagen type 1
Procollagen type 3
Fibrillin
Mucopolysaccharides
Fibroblast growth factor (FGF) receptor 3

Breast milk

(40%)
(60%)

Same
-Breast feeding minimizes the incidence of:
Gynecological tumors: Breast, Ovarian and uterine cancers
Infantile colic
Skin allergy
Asthma
Constipation
Obesity
Diabetes mellitus type1

Cow milk

Genetics and mode of


inheritance
Best diagnostic method
Most common cause of death
Most common causative
organism in cystic fibrosisrelated infection
Pathognomonic hepatic lesion

Most
common
risk factor
Advanced
Maternal
Age over
35 at
delivery

Most
common
type
Nondysjunction

Most Specific
heart defect
atrioventricular
septal defect
(Endocardial
cushion defect)

Cystic fibrosis
-The CFTR gene is located on the long (q) arm of chromosome 7.
-Autosomal recessive.
Sweat chloride test
Respiratory failure
Pseudomonas Aeruginosa

Focal biliary cirrhosis

Downs syndrome (Trisomy 21)


Most common Most
Most common
hematologic
common leukemia
disorder
cancer
Polycythemia
Leukemia -Younger than
one year: acute
nonlymphoblastic
leukemia.
-Older than 3
years: acute
lymphoblastic
leukemia.

Diagnostic method during


pregnancy for Down's
syndrome
Laboratory
Radiological
tests
investigations
-Decreased
Increased
AFP
nuchal
-Decreased
translucency
estriol
in
-Increased
ultrasound.
B-HCG
-Increased
Inhibin
-Increased
PAPPA-A

The diagnostic criteria for Kawasaki disease

Fever for 5 days or more


plus
4 out of 5 of

Polymorphous rash
Bilateral (non purulent) conjunctival injection
Mucous membrane changes, e.g. reddened or dry cracked lips, strawberry tongue, diffuse redness of oral
or pharyngeal mucosa
Peripheral changes, e.g. erythema of the palms or soles, edema of the hands or feet, and in convalescence
desquamation
Cervical lymphadenopathy (> 15 mm diameter, usually unilateral, single, non purulent and painful)

Long and narrow face


Large ears
Large testes
low intelligence quotient
Developmental disability
Autism
Seizures
Attention deficit hyperactivity disorder

Fragile x syndrome characteristics

Fetal alcohol syndrome characteristics

Growth deficiency
Micrognathia
Thin upper lip
Microcephaly
Short palpebral fissures
Not meeting developmental milestones

Pediatrics cardiology
Question
Cyanotic congenital heart diseases

Transposion of great vessels shape


Tetralogy of fallot shape
Complications of ventricular septal defect

Answer

Truncus arteriosus

Transposition of great arteries


Tricuspid valve abnormalities
Tetralogy of fallot
Total anomalous pulmonary venous
connection

Egg shaped silhouette appearance


Boot-shaped appearance
Small
Large

Long-term complication
of unrepaired large VSD

Atrial septal defect findings

Tetralogy of Fallot components

Treatment of hyper- cyanotic spell of Tetralogy of Fallot

Contraindication of Tetralogy of Fallot

-Infective endocarditis
-Pulmonary hypertension
-Right ventricular
hypertrophy
-Right sided heart failure
-Shunt reversal

ASD has a fixed, widely split S2 with a right


ventricular heave and a systolic ejection
murmur. There may also be a mid-Diastolic
rumble.
The commonest presentation is asymptomatic
murmur.
ASD is associated with hypertrophic right
atrium.
Subpulmonary stenosis (most important
determinant of clinical severity and
symptomatology)
VSD
Overriding aorta
Right ventricular hypertrophy
Propranol
Phenylephrine
Knee-Chest position
Fluid
Morphine
NaHCO3
Isoprenaline is inappropriate treatment,
because it reduces systemic vascular resistance
and thus increases the right to left shunt.

Quick revision
Question
Effective management options in children with nocturnal
enuresis (Bed wetting)

Newborns weight changes

Associated metabolic alterations in pyloric stenosis

The most serious complication of perinatal asphyxia


The most suggestive feature of congestive heart failure
in an infant
Neisseria gonorrhea isolated from a child on vaginal
culture is definitive evidence of
Transient hypogammaglobulinemia definition

Infantile colic definition

Shigella dysentariae infection is associated with


Sharp delta waves on the EEG will be indicative of
Holding breath spells is a known precipitant cause of
Blood vessels in the Vitreous is an indicator of
Recommendations of phototherapy in jaundiced
newborn

Breast Milk Jaundice definition

Pathological jaundice definition

Answer
An enuresis alarm
Desmopressin acetate
Imipramin
Behavioral modification
10% decrease in the first days then return to
B.W in the end of first week
B.W doubles in 4-6 months
B.W triples in 12 months
Hypokalemia
Hypochloremia
Dehydration
Alkalosis
Hypoxic ischemic encephalopathy (HIE)
Diaphoresis (Profuse perspiration) with feeding

Sexual abuse
Transient hypogammaglobulinemia is due to exhaustion
of maternally supplied IgG the only ones that cross
placenta. Patient presented with recurrent infection
after 4-6 months of birth.
A benign condition when the baby presented with a
history of crying for 3 hours mainly at evening at 2nd
and 3rd month of age. Management includes
reassurance without medication.
Febrile convulsions
Seizure activity
Generalized convulsions
Prolonged oxygen exposure in preterm neonates
Total serum bilirubin
15
18
20

Timing
25-48 hours old
49-72 hours old
older than 72 hours old

-Jaundice in exclusively breast fed infants in the first


few days of life is due to deprivation of caloric intake,
increasing the breast feeding to more than 10 times per
day is recommended in these patients.
1. Occurs in the first day of life.
2. Or serum bilirubin level increases by >5 mg/dl
per day
or if > 17 mg/dl
or if associated with serious illness.

Fractures related to child abuse


Causes/Association
Abuse in children below 3 years of age
Shaking or jerking of the childs limbs and it is
highly diagnostic of physical abuse in an infant
Squeezing of the chest
Posterior rib fractures
-To roll out shaken baby syndrome by performing an ophthalmologic exam for retinal hemorrhage and a noncontrast CT for subdural hematoma.

Fracture
Spiral fractures of the humerus and femur
Epiphyseal-Metaphyseal Bucket fractures

Notes

Obstetrics and Gynecology

The most common


1
2
3
4
5
6
7
8
9
10
11
12
13
14
15
16

Question
Benign neoplasm of the female genital tract
Gynecological malignancy
Histological type of endometrial cancer
Cause of puerperital pyrexia
Cause of maternal death / cause of post partum death
Cause of post partum hemorrhage
Genital neoplasms occurring in childhood
Ovarian neoplasm during childhood
Type of invasive cancer of cervix
Vaginal infection in women of reproductive age
Immunologic disorder in women of childbearing age
Recognized precipitating factor in spontaneous abortion
Causative organism of mastitis and breast abscess occur in
lactating mother
Protraction or arrest disorder
Medical complication of pregnancy is
Hormonal cause of hirsutism in young women

Answer
Uterine leiomyoma (Fibroids)
Endometrial cancer
Endometrioid adenocarcinoma
Endometritis
Postpartum hemorrhage
Uterine atony
Ovarian cysts and neoplasm
Germ cell tumors
Squamous cell carcinoma
Bacterial vaginosis
Idiopathic thrombocytopenic purpura
Chromosomal abnormalities
Staphylococcus aureus
Cephalopelvic disproportion
Pyelonephritis
Excess androgen production by
noncycling ovaries

Uses of Magnesium sulphate


-Treatment of preterm labor
-prevention of seizures in patients
with preeclampsia

Uses of Ergotamine
-Treatment of acute migraine attacks
-Prevent post-partum hemorrhage

Advantage of mediolateral episiotomy


-Less likely to cause a fourth degree extension

Advantages of median episiotomy


-Easier surgical repair of the episiotomy
-Improved healing of the episiotomy
-Less blood loss
-Less pain
Tamoxifen

Advantages
-Hormonal adjunctive treatment has proven
benefits in pre and post menopausal women.
-Anti-estrogen used for treatment of breast cancer,
and tumors that express estrogen/progesterone
receptors respond better to it.
-It results in better survival regardless of tumor
staging or grading.

Disadvantage
-Paradoxical tamoxifen-induced endometrial
hyperplasia.

Finding

Association

Early deceleration
Variable deceleration
Late deceleration
(Most serious and dangerous)

Head compression
Umbilical cord compression
Uteroplacental insufficiency/Fetal hypoxia

Placenta previa
-Painless vaginal bleeding >28 weeks

Placenta abruption
-Painful 3 trimester vaginal bleeding + contraction
- (+/- Fetal distress)
-The most appropriate diagnostic method for both Placenta previa and Placenta abruption is
transabdominal ultrasound.
rd

Ultrasonographic Prenatal assessment of gestational age


-First-trimester crown-rump length is the best parameter for determining gestational age.
-In the second and third trimesters, estimation of gestational age is accomplished by measuring the
biparietal diameter, head circumference, abdominal circumference, and femur length.

Labs
-High LH
-High FSH
-High Androgen
-High Estrogen
-Low progesterone

polycystic ovarian syndrome


Complications
Women with polycystic ovarian
syndrome are at great risk of
developing;
1-Type 2 Diabetes
2-Dyslipidemia
3-Endometrial cancer

Treatment
-Metformin and clomiphene
alone or in combination are firstline agents for ovulation
induction.
-Spironolactone will improve
hirsutism and menstrual
irregularities.

Gestational Diabetes
Fetal complication
1-Fetal Macrosomia; operative delivery risk (Cesarean
section), birth Trauma risk and shoulder dystocia.
2-Metabolic; hypoglycemia, hypocalcaemia and
hypoglycemia.
3-Hypothermia
4-Hyperbilirubinemia
5-Caudal regression syndrome
6-Premature birth
7-Respiratory distress syndrome
8-Hematologic; Polycythemia, hyperviscosity,
thrombocytopenia and low iron stores.
9-Obesity during childhood
10-Cardiomyopathy
11-Congenital anomalies
12-Perinatal asphyxia

Maternal complication
1-Longterm risk of developing Diabetes Mellitus: 50%
2-Hypertension
3-Preeclampsia
4-Ceserean Section

o
o
o
o

The initial evaluation for chronic pelvic pain


Urinalysis and culture
Cervical swabs for gonorrhea and Chlamydia
CBC
Erythrocyte sedimentation rate
B-hCG level
pelvic Ultrasonography

Effective management of Hot flashes


Estrogen replacement therapy.
Escitalopram(Cipralex): an antidepressant of the selective serotonin reuptake inhibitor (SSRI) class
Infertility in women
The most likely cause of infertility in a normally menstruating woman under the age of 30 is Pelvic
inflammatory disease.
The most likely cause of infertility in a normally menstruating woman over the age of 30 without
a history of Pelvic inflammatory disease is endometriosis.
The most likely cause for infertility in a woman under 30 with abnormal menstruation is
polycystic ovary syndrome.
The most common preventable cause of infertility overall is Pelvic inflammatory disease.
Ectopic pregnancy
The most common site is fallopian tube
mostly in the Ampulla.
The 3 most common presenting complaints are: amenorrhea, abdominal pain and vaginal
bleeding.
The most appropriate initial step for diagnosis is serum HCG.
Culdocentesis is a diagnostic procedure.
Laparoscopy is a confirmatory procedure.

Gestational age
The recommended time to screen for gestational diabetes is 24-28 weeks.
Amniotic Fluids reach it maximum Volume (1 liter) at 34-36 weeks.
The definition of post-term pregnancy is a pregnancy that has reached 42 weeks gestation.

-Definition
-Treatment

Lichen sclerosus
A chronic, progressive, inflammatory skin condition found in the anogenital region. It is
characterized by intense vulvar itching.
High-potency topical corticosteroids

Drugs
NSAIDs (e.g. Naproxen)
Combined Aspirin and heparin
Low molecular weight heparin
Atenolol and Propranolol
Alpha-Methyldopa, Hydralazine, and
Labetalol
Methotrexate
(Cytotoxic drugs)
Trimethroprim/Sulfamethaxazole
Nitrofurantoin

Pregnancy and pharmacology


Comments
Most appropriate choice of management of primary dysmenorrhea.
Used to treat pregnant women with antiphospholipid syndrome.
The preferred anticoagulant for venous thrombosis during
pregnancy.
Associated with intrauterine growth retardation when used for
prolonged periods during pregnancy.
Most commonly treatment used for gestational hypertension.

An absolute contraindicated drug in breast feeding.

Effective treatment for acute uncomplicated cystitis in women.


Effective treatment for acute uncomplicated cystitis in women who
have allergy to sulfa drugs.
The drug of choice for treatment of Chlamydia trachomatis
infection in non-pregnant women.
The drug of choice for treatment of Chlamydia trachomatis
infection in pregnant women
Considered first-line treatment for a female patient presented with
multiple painful genital vesicles.
The treatment of choice of endometriosis.
Anti-estrogen used to increase FSH levels and induced ovulation in
infertile patients.
The best medication to be given for gestational diabetes mellitus.
A dopamine agonist that is used in the treatment of
hyperprolactenemia.
Side effect : postural hypotension
The most effective and the most common method of postcoital
contraception.
Emergency contraception does not interfere with an established,
post implantation pregnancy.
Contraceptive method.
It is not causally linked with thromboembolic events. So, it is
suitable for patients with sickle cell disease.
Considered first-line treatment for premenstrual dysphoric disorder
and postpartum depression.
The most appropriate antibiotic used to treat Trichomonas vaginalis
and bacterial vaginosis.
The most appropriate pharmacotherapy for Rh negative pregnant
mom.
Contraindicated during pregnancy.

Doxycycline

Azithromycin

Acyclovir (Oral)

Danazole
Clomiphen

Insulin
Bromocriptine

Norgestrel/Ethinyl estradiol (Ovral)

o
o
o

Injectable medroxyprogesterone
acetate (Depo-Provera)

o
o

SSRIs

Metronidazole

RhoGAM (anti-D immunoglobulin)

Angiotensin-converting-enzyme
inhibitor

Type
Complete
Incomplete
Inevitable

Threatened
Missed abortion
Septic abortion

Abortion
Definition/ US findings
No products of conception found.
Some products of conception found.
Products of conception are intact but there is intrauterine bleeding and
dilation of cervix.
Products of conception intact, +ve bleeding, no cervical dilation.
Death of fetus, but all products of conception are present in the uterus.
Infection of the uterus and the surrounding area.

Pregnancy and Heart Disease


The most common cause of deterioration of cardiac status in a pregnant woman with rheumatic
heart disease is mitral stenosis.
Severe mitral regurgitation and rheumatoid arthritis are tolerable during pregnancy.
The most dangerous heart disease in pregnant women is: peripartum cardiomyopathy, then
Eisenmenger syndrome.

postpartum
psychiatric illness

Maternity blues

Postpartum depression

POSTPARTUM PSYCHOSIS
(Most severe form)

Onset

Postpartum blues develop


on the third or fourth
postnatal day. Symptoms
resolve within days.

Postpartum depression
tends to start within a
month of delivery. Many
cases resolve within few
weeks.

Postpartum psychosis
develops usually in the
second postnatal week.

Disease
Association

Most useful
diagnostic procedure

Hydatidiform mole
Absence of fetal heart sound.
Increase fundal height more than expected.
Vaginal spotting.
Vomiting.
Proteinuria.
Increase level of B-hCG.
Ultrasound

Quick revision
The most appropriate diagnosis for a woman presented with greyish fouly smelling acidic vaginal
discharge (PH<4) associated with Itching, and wet smear showed presence of clue cells is
bacterial vaginosis.
The most likely diagnosis of a pregnant women presented women presented with sudden onset
of lower abdominal pain, fainting, and the presenting part cannot be felt on vaginal examination
is uterine rupture.
In the symptomatic patient with uterine fibroids unresponsive to medical therapy, myomectomy
is recommended over fibroid embolization for patients who wish to become pregnant in the
future.
Melasma or chloasma is common in pregnancy, with approximately 70% of pregnant women
affected. It is an acquired hypermelanosis of the face, with symmetric distribution usually on the
cheeks, nose, eyebrows, chin, and/or upper lip.
Treatment of asymptomatic Chlamydia trachomatis infections in women reduces their risk of
developing pelvic inflammatory disease, tubal infertility, ectopic pregnancy, and chronic pelvic
pain.
Thyroid function must be evaluated in women with postpartum depression.
The criteria for severe preeclampsia specify a blood pressure of 160/110 mm Hg or above on two
occasions, 6 hours apart. Other criteria include Proteinuria above 5 g/24 hr, thrombocytopenia
with a platelet count<100,000/mm3, liver enzyme abnormalities, epigastric or right upper
quadrant pain, and alteration of mental status.
The risk of breast cancer is increased by late pregnancy.
Carcinoma of cervix is associated with HPV subtype 16, 18, 31, 45.
Hyperemesis gravidarum is associated with increased liver enzymes in 25% of cases.
Consumptive coagulopathy (DIC) is one of complications of IUFD after 4 weeks.
Vaginal hysterectomy is contraindicated in menstruating women.
Active pelvic inflammatory disease is an absolute contraindication for IUD.
Glycosuria may develop in the normal pregnancy because there is an increase in glomerular
filtration rate of glucose without an increase tubular reabsorption.
Papillary projection of ovarian cyst in ultrasound suggests the need for immediate laprotomy.
The initial step in evaluation of a woman of childbearing age with a history of abdominal pain and
vaginal spotting is pregnancy test (urine or serum B-hCG).
Persistent and recurrent vaginal candidiasis (Monilial Vaginitis) is associated with DM.
Recurrent vaginal infection in prepubertal girls mainly due to mixed normal flora.
Histopathology of papillary serous cystadenocarcinoma (Common form of malignant ovarian
cancer) shows psammoma bodies.

Management of preterm labor includes


Tocolytic(Magnesium sulphate)
Antibiotic(Penicillin: prevention of GBS)
Corticosteroid(Dexamethasone: reduction
of RDS,IVH and perinatal mortality)

Management of severe preeclampsia


Hydralazine
Magnesium sulphate
Delivery
Labor induction with IV
Oxytocin
Note: Prior to 32 weeks
Expectant management and administration
of glucocorticoids

Amenorrhea: absence of menstrual periods, either on a permanent or temporary basis.


Type
Primary amenorrhea
Secondary amenorrhea
Definition
Menstrual periods have never
a-Previously regular cycles: 3
begun (by age 16)
months of no Menses
b-Previously irregular cycles: 6
months of no Menses
Most common causes
-The most common cause of
-The most common physiologic
primary amenorrhea is Turner's
cause of secondary amenorrhea
syndrome.
is pregnancy.
-The most common anatomic
cause of secondary amenorrhea
is Ashermans syndrome.

Oral contraceptives
Benefits
On the other hand
-Reduction the risk for carcinoma of the ovary,
- Oral contraceptives do not reduce the risk of
endometrium and colorectum.
carcinoma of the breast, cervix, lung, or head and neck.
- Reduction the risk of postmenopausal fractures.
-The risk of thromboembolism is increased.
Oral contraceptive pill (OCP) is contraindicated in
May lead to VACTERL (a malformation complex of Vertebral, Anal, Cardiac, Tracheo Pregnancy
Esophageal, Renal and Limb anomalies)
This is because the use of oral contraceptives may diminish the quality and quantity of
Lactation
milk in the post-partum period.

Notes

Surgery

The most common


Question

Answer

1
2
3
4
5
6
7
8
9
10
11
12

Cause of necrotizing fasciitis

Group A Streptococcus

Causative organism of furuncle


Cause of bloody nipple discharge
Histological type of breast cancer
Site of metastasis of carcinoma of breast
Site of metastasis of carcinoma of colon
Hernia
Side of Indirect inguinal hernia
Hernia that prone to be incarcerated
Causes of hemorrhagic cystitis

Staphylococcus aureus
Benign Intraductal papilloma
Infiltrating ductal
Spine
Liver
Indirect inguinal hernia
Right side
Femoral hernia
Cyclophosphamide and Radiation

Cause of urinary incontinence in elderly

Detrusor instability

Mode of presentation of Prostatic cancer nowadays

13
14
15
16
17
18
19
20
21
22
23
24
25
26

Initial sign of testicular cancer


Complication post Transurethral resection of the prostate
Cause of fever first day post operative
Cause of hyponatremia in a post operative patient
Site of stone entery in "gallstone ileus"
Cause of acute mesenteric ischemia

Elevated PSA and/or Abnormal


findings on digital rectal examination
Painless testicular enlargement
Hypokalemia
Atelectasis
Excess water
A cholecystoduodenal fistula
Superior mesenteric artery embolism

Location of appendix
Type of gall bladder stone

Cause of death during vascular surgery


Parotid malignancy
Parotid benign tumor
Cause of constipation
Site of duodenal ulcer
Location of anal fissure

Most common renal stone


Calcium oxalate
Radio-opaque

Retrocecal

Mixed
Myocardial infarction
Mucoepidermoid carcinoma
Mixed tumor
Intestinal obstruction
First part of duodenum
Posteriorly in the midline

Second most common renal stone


Uric acid
Radiolucent

Evaluation of thyroid nodule


First step
Low TSH
Normal or high TSH
Second step
A radionuclide thyroid scan
A fine-needle aspiration biopsy of the nodule
-Most important diagnostic test should be carried out for a patient presented with thyroid nodule is fine needle
aspiration.

Intestinal pseudo-obstruction (i.e. no mechanical lesion but clinical and radiological features of
obstruction) may be a manifestation of:
well recognized causes:
rarely, it is the presenting manifestation of occult
neoplasm:
Amyloidosis
Small cell lung carcinoma
Lead poisoning
Pheochromocytoma
Parkinsons disease
Progressive systemic sclerosis
Diabetes mellitus
Hypothyroidism
Chronic phenothiazine therapy
-Conditions lead to chronic overflow incontinence
caused by obstruction at the bladder neck or a
hypotonic bladder caused by autonomic neuropathy can
result in hydronephrosis and impaired renal function
1- Benign prostatic hypertrophy
2- DM
3- Hypothyroidism
4- Uremia
5- Collagen vascular disease
6- Guillain-Barre syndrome

-Conditions lead to automatic bladder emptying

1- Multiple strokes
2- Alzheimer's disease
3- Brain tumors
4- Normal-pressure hydrocephalus

Term

Meaning

Modified radical mastectomy

Removing of breast, axillary nodes (1, 2) and nipple


areolar complex. Pectoralis major and minor are
not removed. Drains are placed to drain lymph
fluid.

Proctalgia fugax (Levator Syndrome)

An episodic, sudden, sharp pain in the anorectal area


lasting several seconds to minutes.

Boil (furuncle)

An acute deep infection of hair follicle (folliculitis).

Voiding symptoms
Irritative
-Urgency
-Dysuria
-Frequency
-Nocturia

Obstructive
-Hesitancy
-Decreased force of stream
-Intermittency
-Postvoiding dribbling

Diagnostic or radiological tool


Question
The most accurate diagnostic tool for acute appendicitis
The most appropriate diagnostic step for renal cell carcinoma
The most sensitive and specific radiological test for diagnosis of diverticulitis

The best initial radiological test for a patient presented with renal colic
The most appropriate initial test done in a patient with hematuria
The most effective method of screening for colon cancer
The most sensitive and least expensive method to discriminate
between testicular and extratesticular masses
The most useful tool to roll out bladder injuries in trauma patients
The most useful tool to detect early bladder cancer

Answer
CT scan
CT scan
CT scan
KUB
Intravenous pyelogram
Colonoscopy
Ultrasound Doppler
Retrograde cystogram
Cystoscopy

The definitive in a prostate cancer


The definitive diagnostic method in Hirschsprung disease

Prostatic biopsy
Rectal biopsy

The most appropriate step in diagnosis of post operative pulmonary


thromboembolism
The definitive diagnostic method of post operative pulmonary
thromboembolism

Spiral CT scan

Laboratory marker
AFA
CEA
5 HIAA

Classification of thyroid cancer


Papillary
Follicular
Medullary
Anaplastic

Pulmonary
angiography

Indication
Hepatocellular carcinoma
Metastatic colon cancer
Carcinoid tumor

Comment
The commonest type and has excellent prognosis, metastasize
mainly through lymphatics route.
The second most common, metastasize mainly through
hematogenous route.
The third most common and the most likely having familial
tendency, originates from the parafollicular cells (C cells).
The worst prognosis.

Disease

Radiological findings

Hirschsprung disease

Multiple dilated small


loops of small bowel
and microcolon.

The most appropriate


initial step in
management
Diverting ileostomy (to
relieve the functional
obstruction) and
appendectomy (to obtain
tissue for pathology).

The definitive
treatment
Total colectomy

Types of germ cell tumor

sign
Cullen's sign

Grey turner's sign


Kher's sign
Balance sign

Seatbelt sign

Teratoma/Dermoid cyst ( most common)


Sertoli-Ledyding cell tumor
Granulosa theca cell tumor

Indication
Bruising around umbilicus and it indicates hemorrhagic
pancreatitis or ruptured abdominal aortic aneurysm.
Bruising in the flank and it indicates retroperitoneal hemorrhage.
Pain in the left shoulder and it indicates splenic rupture.
Dull percussion of the left and shifting dullness on the right and it
indicates splenic rupture.
Bruising where seatbelt was, and it indicates deceleration injury.

Factors cause elevated results on prostate-specific


antigen (PSA) screening
Performance of prostate massage
Performance of cystoscopy
Urinary tract infection
Urinary retention

Factor does not elevate results on prostate-specific


antigen (PSA) screening
Performance of a rectal examination

Burn
Rule of 9's for Adults: 9% for each arm, 18% for each leg, 9% for head, 18% for chest, 18% for
back and 1% for perineum.
Rule of 9's for Children: 9% for each arm, 14% for each leg, 18% for head, 18% for chest, 18% for
back and 1% for perineum.
Fluid Requirements = Total body surface area burned(%) x Wt (kg) x 4mL
Give 1/2 of total requirements in 1st 8 hours, and then give 2nd half over next 16 hours.
The initial management of an adult patient presented with major burn is IV crystalloid.

Indirect inguinal hernia


-More frequent (85%).
-Patient is usually young, and it is about 20 times
more common in male than female, and nearly one
third are bilateral.
-Hernia may reach the scrotum or labium majus,
because it passes through the deep inguinal ring,
traverses the inguinal canal and comes out through
the superficial inguinal ring.
-The hernia sac is elongated and lies in front of the
spermatic cord.
-The neck of the sac is narrow.
-The hernia sac is the remain of the processes
vaginalis
Congenital origin.

Direct inguinal hernia


-Less frequent (15%).
-Patient is usually old, and it is about 8 times more
common in male than female, and most cases are
bilateral.
-Hernia comes forward through the posterior wall
of the inguinal canal medial to the inferior
epigastric vessels.
-The hernia sac is globular and lies behind the
spermatic cord.
-The neck of the sac is wide.

Inguinal hernia
Hernia bulges above the inguinal ligament.
Hernia sac is above the pubic tubercle.

Hernia is more common in males


(Well developed inguinal canal)

Femoral hernia
Hernia bulges below the inguinal ligament.
Hernia sac is below and lateral the pubic
tubercle.
Hernia is more common in females.
(Wider femoral ring)

Condition
Testicular torsion

Epididymo-orchitis
Hydrocele

Varicocele

Testicular cancer

o
o
o
o
o
o
o
o
o
o
o
o
o
o
o
o
o

Features
Acute pain and swelling (Emergency case).
Affected testis typically lies higher.
Primarily affects teenagers.
Abnormal cremastric reflex.
Acute pain, nausea and fever.
Pain relief upon elevation of scrotum.
Transilluminates.
Sometimes associated with testicular tumors.
Insidious, painless.
Contorted, dilated veins.
Feels like a bag of worms.
May cause infertility.
Empties with patient in supine position.
Insidious, painless.
Usually painless mass.
Sometimes may resemble epididymitis.
Spread very easily especially during vigorous
palpation or biopsy.

Over (abnormal) healing


Hypertrophic scar
Keloid scar
o Hypertrophic scars develop soon after
o Keloid scars may not develop for several
injury; they are confined to the wound
months, but rarely subside and often
and usually subside over time.
progress.
o They favor flexor surfaces and are
o Keloids overgrow the boundaries of the
common over the shoulder joint.
injury and are prone to develop on the
ear lobes and the pre-sternal skin.
o In contrast to hypertrophic scars, the
magnitude of a Keloid scar is not
proportional to the magnitude of the
injury.
o Keloids recur after surgical excision, but
this may be prevented by preoperative
radiotherapy or intradermal steroid
injections.

Fibrocystic change
Often bilateral
Multiple nodules
Menstrual variation
May regress during pregnancy
Fibrocystic change does not increase risk of
breast cancer, but makes detection more
difficult

Breast cancer
Often unilateral
Single mass
No cyclic variations

Cancer statistics in both genders


Incidence
Male

Female

1. Prostate
2. Lung
3. Colorectal

1. Breast
2. Lung
3. Colorectal
Mortality

Male
1. Lung
2. Prostate
3. Colorectal

Female
1. Lung
2. Breast
3. Colorectal

Radiology and surgery

Pulmonary contusion produces classic "white-out" of lungs on CXR, where the contused lung is
exquisitely sensitive to fluid overload and the fluid leaks easily along with signs of respiratory distress.
Pneumatosis intestinalis is a radiological sign which is highly suggestive for necrotizing enterocolitis.
Corkscrew appearance is associated with malrotation with midgut volvolus/ and esophageal spam.
The double bubble sign with a little gas beyond is highly suggestive of malrotation.

Case

Diagnosis

Congenital cystic dilatations of the biliary tree associated with jaundice, fever,
and intermittent abdominal pain, an abdominal mass and direct
hyperbilirubinemia
The most probable diagnosis of a patient presented with pallor, paraesthesia,
and pulsnessless of lower limb
The most probable diagnosis of a female presented with firm, movable, rubbery
breast mass

Choledochal cysts

The most likely cause of recurrent painful swelling of the submandibular


gland

Calculi

Arterial thrombosis
Fibroadenoma

Quick revision

Villous adenomas are the most likely to develop into adenocarcinoma.


Familial adenomatous polyposis (Autosomal dominant type) carries 100% risk for colorectal cancer.
The treatment of choice of thrombosed external piles is local anesthesia and thrombectomy.
Seminoma is a radiosensitive and highly curable tumor. The main risk factor is cryptochidism.

Bloody fluid obtained by fine-needle aspiration of a breast nodule (simple cyst) must be sent
for cytological examination.

Peritonitis is an indication of diagnostic peritoneal lavage in trauma victim.


Borders of Hesselbach triangle are inferior epigastric vessels, rectus sheath and inguinal ligament.
The inguinal ligament is formed by the lower most border of external oblique aponeurosis.
The most important early sign of wound infection is pain at the wound.
The best indicator for adequate IV fluid is adequate urinary output.
In acute appendicitis, rigidity may be absent if the appendix is pelvic in position.
A cold abscess is an abscess that usually accompanies tuberculosis commonly in the lymph nodes and
bone.
Excision rather than bypass is preferred for surgical treatment of small intestine Crohn's disease,
because it reduces small intestine cancer.

Pharmacology

Starting an Alpha-blocker (tamsulosin (Flomax)) at the time of insertion of the urethral catheter has
been shown to increase the success of a voiding trial.
Initiation of statins (lipid lowering drugs) prior to vascular surgery (e.g., iliofemoral bypass) have been
shown to have plaque-stabilizing and vascular anti-inflammatory effects and it significantly reduces
cardiovascular risk.
Fiber supplementation for patients with symptomatic hemorrhoids is very helpful in decreasing
bleeding, pain, prolapse and itching.

Notes

Orthopedic surgery

The most common


1
2
3
4
5
6
7
8
9
10
11
12
13
14
15
16

Question
Cause of pathologic fracture
Causative organism of acute infectious (septic)
arthritis overall in adults and children
Causative organism of acute infectious (septic)
arthritis in young, sexually active adults
Route of acute infectious (septic) arthritis
Joint affected in acute infectious (septic) arthritis
Nerve injury associated with fracture of humerus
Cause of joint disease
Cause of hand infection
Cause of avascular necrosis
Location of intervertebral disc herniation
Cause of pathologic fracture
Bacterial causative organism of osteomyelitis
overall
Bacterial causative organism of osteomyelitis in
sickle cell disease patients
Bacterial causative organism of osteomyelitis in
diabetic patients who develop a diabetic foot
Finding of osteomyelitis
Dislocated joint in children

Answer
Osteoporosis
Staphylococcus aureus
Neisseria gonorrhoeae
Hematogenous
Knee
Radial nerve
Osteoarthritis
Trauma
Trauma
L5-S1
Osteoporosis
Staphylococcus aureus
Salmonella
Polymicrobial infection
Pain over the involved area of bone
Elbow

Bone tumors
The most common type of bone tumor
The most common primary bone cancer
The second most common primary bone cancer

Diagnostic or radiological tool


Question
The most effective imaging study for diagnosing osteomyelitis
and assessing the extent of disease process
The most direct and accurate diagnostic method of
osteomyelitis
The most appropriate next step in management of a patient
presented with a history suggestive of septic arthritis
The most sensitive and specific way of investigating patients
with Avascular necrosis
The best diagnostic test for osteoporosis

Metastatic
Osteosarcoma
Ewing's sarcoma

Answer
MRI
Needle aspiration of infected
bone or bone biopsy
Diagnostic arthrocentesis
(synovial fluid analysis)
MRI
A central DXA scan of the hip,
femoral neck, and lumbar
spine

Laboratory markers
Question
Major circulating form of vitamin D
The most important regulator of serum
1,25(OH)2 Vitamin D
The initial parameter to assess response to
Vitamin D supplements

Symptoms and signs

Early treatment
(Within 48 hours of
onset)
Late treatment
(After 48 hours)

- Surgical drainage and antibiotics

Most probable diagnosis


Most appropriate management

Signs

First line treatment

Raised serum Phosphorus then raised serum


Calcium

Pyogenic tenosynovitis
Progressively worsening pain in the index finger.
On examination the finger is swollen and held in a flexed position. The
pain increases with passive extension of the finger, and there is
tenderness to palpation from the tip of the finger into the palm.
-Antibiotics and splinting

History

Symptoms

Answer
Serum 25-hydroxyvitamin D
Parathyroid hormone

A patient presented with maximum tenderness in the area of


the anatomic snuff box after falling on his outstretched hand.
Scaphoid fracture.
A thumb spica splint and follow-up radiographs in 2 weeks.

Plantar fasciitis
Typically the pain is located in the plantar surface of the heel and is worst when
the patient first stands up when getting out of bed in the morning (first step
phenomenon) or after prolonged sitting. The pain may then improve after the
patient walks around, only to worsen after prolonged walking.
Typical findings include point tenderness to palpation on the plantar surface of
the heel at the medial calcaneal tuberosity where the calcaneal aponeurosis
inserts.
Over-the-counter heel inserts

Alternatives to the bisphosphonates(Alendronate)


Medications
Advantages
Intravenous zoledronic acid
Reduce both hip fracture risk and
vertebral fracture risk
Teriparatide, raloxifene and Calcitonin-salmon
Reduces vertebral fracture risk but not hip
fracture risk

Quick review
Question
Typical radiographic findings of osteoarthritis
include
Typical examination findings of osteoarthritis
are
The most frequent ligament injury among
traumatic knee injuries
Mechanism of medial collateral ligament sprain
Presentation of medial collateral ligament
sprain

Maneuver for detecting meniscal tears


Maneuvers for detecting an anterior cruciate
ligament (ACL) tear
The strongest risk factor for osteoporosis
Saturday night palsy definition

Benefit of vitamin D in elderly


Management of femur fracture

Pathognomonic diagnostic sign for carpal


tunnel syndrome
Types of Epicondylitis

Types of limp
Antalgic
Trendelenburg
High stepping gait
Spastic
Ataxic

Answer
-Joint space narrowing
-Osteophytes
-Subchondral bone sclerosis and pseudocysts
-Joint crepitus
-Joint swelling
-Limited range of motion
Medial collateral ligament sprain
It results from to lateral blow to the joint
The patient may present with tenderness along
the median aspect of the joint line and
increased laxity in a valgus stress with flexed
knee at 30 degrees.
McMurrays test is used to detect meniscal
tears
The most accurate maneuver for detecting an
ACL tear is the Lachman test, followed by the
anterior drawer test and the pivot shift test.
Age
An acute radial mononeuropathy which results
from compression of the radial nerve at the
spiral groove
Vitamin D decreases the risk of fall in elderly
Intratrochantric fracture
(extracapsular): closed reduction and
internal fixation
Intracapsular: prosthetic femoral head
replacement
Tinels sign

Lateral Epicondylitis (Tennis elbow)


Medial Epicondylitis (Golfers elbow)

Pathology
Hip joint arthritis
Weakness of hip abductors
Foot drop secondary to common peroneal nerve palsy
Cerebral palsy
Cerebellar pathology

Pediatric Orthopedics
The most probable diagnosis of 6-years old boy presented with a history of insidious
development of limping with decreased motion in one hip, occasional ipsilateral knee
pain and antalgic gait is avascular necrosis of the capital femoral epiphysis (Legg-CalvePerthes disease).
The most probable diagnosis of a choppy 12-14 years old boy presented with hip pain
and inability to internally rotate the hip is Slipped capital femoral epiphysis. The
radiological sign is "melting ice cream cone".
The most common cause of knee pain in pediatric age group is OsgoodSchlatter disease.
Elbow fracture in pediatric age group
-The most common type
Supracondylar fracture
-The most serious complication
Volkmann Ischemic contracture (Compartment syndrome)
-Management
Manipulation, reduction, and immobilization of the arm at
90 degrees with slight pronation.

Nerve
Radial

Motor deficit
Wrist extension

Sensory deficit
Dorsal forearm
and hand
(The first 3 fingers)
Palmar surface
(The first 3 fingers)

Common cause
Humeral fracture

Clinical findings
Wrist drop

Median

-Pronation
-Thumb opposition

Carpal tunnel
syndrome
Elbow dislocation

Abduction

Palmar and dorsal


surface
(The last 2 fingers)
Lateral shoulder

Weak wrist flexion


and flat thenar
eminence
Claw hand

Ulnar

Finger abduction

Axillary

Peroneal

-Dorsiflexion
-Eversion

Dorsal foot and


lateral leg

Knee dislocation

Anterior humeral
dislocation

-Numbness over
the deltoid muscle
on the outer upper
arm.
-Difficulty raising
the arm out to the
side.
-Wasting of the
deltoid muscle in
prolonged injuries.
Foot drop

Notes

Otorhinoloryngology

The most common


1
2
3
4
5

Question
Site of malignancy in paranasal sinus
Cause of otorrhea
Cause of chronic cough
Cause of acute sinusitis
Sinuses involved in sinusitis

Answer
Maxillary sinus
Acute otitis media
Post nasal drip
Viral
Maxillary sinuses

Mnires disease
Mnires disease (also called endolymphatic hydrops) is a long term, progressive condition affecting the
balance and hearing parts of the inner ear.
Presentation: Triad of (Tinnitus, Vertigo and Sensorineural Hearing Loss) associated with (Headache, Ear
pressure and Nystagmus)

Quick revision
Question
The most likely diagnosis of a child presented with
dysphagia and increasing sore throat and soft tissue
swelling of the posterior pharyngeal wall post upper
respiratory tract infection
The most probable diagnosis of a child presented with
difficulty of swallowing and deviated uvula
The most probable diagnosis of a child presented with
foul-smelling unilateral nasal discharge
A condition of rebound nasal congestion brought on by
extended use of topical decongestants (e.g.,
Oxymetazoline, Phenylephrine, Xylometazoline, and
naphazoline nasal sprays) that work by constricting blood
vessels in the lining of the nose
The most sensitive physical finding for otitis media

Pharmacologic measures

Nonpharmacologic measures

Answer
Retropharyngeal abscess

Peritonsillar abscess
Foreign body
Rhinitis medicamentosa

Tympanic membrane immobility

Allergic rhinitis
Intranasal corticosteroids are the most effective treatment
for mild to moderate allergic rhinitis and should be first-line
therapy.
Second-line therapies that can be used for symptoms that do
not respond to initial treatment include antihistamines,
decongestants, cromolyn, and leukotriene receptor
antagonists.
Nasal irrigation and avoiding irritants.
The most and best desirable treatment for nasal allergy is
allergen avoidance.

Notes

Ophthalmology

The most common


Question
1 Cause of uncorrectable visual loss in children
2 Cause of Amblyopia
3 Cause of visual loss in elderly
4 Cause of preventable blindness worldwide
5 Cause of red eye
6 Cause of conjunctivitis
7 Virus associated with conjunctivitis
8 Bacteria associated with conjunctivitis
9 Type of Glaucoma
10 Cause of spontaneous vitreous hemorrhage
11 Cause of Pupillary dilatation

Findings
Bitemporal hemianopia
Retrobulbar neuritis
Intraocular opthalmoplegia
Optic neuritis

Kayser-Fleischer ring
Argyll Robertson Pupil
Lens dislocation

Leukocoria (absent red reflex)


Roth spots
Copper wiring, flame hemorrhages, A-V nicking
Papilledema
Cherry-red spot on macula

Answer
Amblyopia
Strabismus
Macular degeneration
Chlamydia trachomatis infection
Conjunctivitis
Viral
Adenovirus
Streptococcus pneumoniae
Open angle Glaucoma
Proliferative diabetic retinopathy
Compression of oculomotor nerve

Associated diseases/causes
Usually caused by a pituitary tumor
Initial sign of multiple sclerosis
Classically found in multiple sclerosis
-Viral infection
-Multiple sclerosis
-Vasculitis
-Methanol
-Meningitis
-Syphilis
-Tumor metastasis
Pathognomonic for Wilsons disease
Pathognomonic for tertiary syphilis (neurosyphilis)
Upward: Marfans syndrome
Inferiorly: Homocystinuria
Variably: Alports syndrome
Retinoblastoma
Endocarditis
Subacute hypertension
-Atherosclerosis
Increased intracranial pressure
-Tay-Sachs disease
-Niemann-pick disease
-Central retinal artery occlusion

Retinal conditions
Senile macular degeneration
Retinal detachment
Optic neuritis
Central retinal artery
Central retinal vein occlusion

presentation
Slow painless loss of central visual acuity
Painless blurred vision
Painful sudden loss of vision
Painless sudden loss of vision
Painless gradual loss of vision

-Glaucoma: Increased Intraocular Pressure with Optic Nerve injury


-Major cause of blindness in the aging and leading cause of blindness in African-Americans.
Type
Information
presentation
Medical treatment
surgical treatment
Open Angle
Glaucoma

- Loss of vision can


be largely prevented
by regular eye
check-ups
(measurement of
ocular pressure )

-Asymptomatic
until late stages.
-Gradual loss of
peripheral vision
(tunnel vision) is
the earliest
presenting
symptom.
-The earliest
finding (sign) of
open angle
Glaucoma is
elevation in
intraocular
pressure.

-Prostaglandin
Analogues.
-Intraocular Beta
Blockers (Timolol).
-Intraocular
Cholinergics
(Pilocarpine).
-Intraocular
Sympathomimetics.
-Topical Carbonic
Anhydrase Inhibitor
(acetazolamide).
- Intraocular AlphaAdrenergic
(Apraclonidine).

Laser
trabeculoplasty

AngleClosure
Glaucoma

- It is a medical
emergency.
-Due to blockage of
aqueous drainage.
-May be precipitated
by pupil dilation
(Mydriatics;
atropine)

-Rapid onset.
-Severe pain.
-Blurred vision.
-Halos around
lights.
-Redness.
-Fixed mid-dilated
pupil.

-Systemic Carbonic
Anhydrase Inhibitor
(acetazolamide).
-Intraocular Beta
Blockers (Timolol).
- Intraocular AlphaAdrenergic
(Apraclonidine).
-Intraocular
Cholinergics
(Pilocarpine).

Laser iridotomy

-The commonest cause of intranuclear opthalmoplegia (INO) in young patient is multiple sclerosis.
-INO: abnormal horizontal ocular movements with lost or delayed adduction and horizontal Nystagmus of
the abducting eye.
-Optic neuritis is the initial symptoms in approximately 40% of persons who eventually are diagnosed with
multiple sclerosis.
-Optic neuritis is associated with partial or total loss of vision, pain on motion of the involved eye,
scotoma affecting macular vision, and a variety of other visual-field defects.
The tonometer's usage contraindications are the following:
-upper lid pathology (inflammatory diseases, scars, eyelid deformation);
-Expressed sclera and/or conjunctiva pathology in the measuring area.

Diabetic retinopathy
-Non-proliferative:
-Microaneurysms
-Dot and blot hemorrhages
-Flame-shaped hemorrhages
-Hard exudates
-Cotton-wool spots (soft exudates)
-Venous loops and venous beading
-Intraretinal microvascular abnormalities
-Macular edema
-Proliferative:
-Neovascularization
-Preretinal hemorrhages
-Hemorrhage into the vitreous
-Fibrovascular tissue proliferation
-Traction retinal detachments
-Macular edema

Hypertensive retinopathy
Keith Wagener Barker (KWB) Grades
Grade 1

Copper wire arterioles

Grade 2

As grade 1 + Vein indentations


at a-v crossing or nicking or
nipping

Grade 3

As grade 2 + with cotton wool


spots and flame-hemorrhages

Grade 4

As above but with swelling of


the optic disk (Papilledema)

In autoimmune thyroid eye disease


The characteristic finding in thyroid eye disease is inflammatory swelling of the extraocular
muscles.
Impairment of upward gaze in the abducted eye, because inferior rectus is most commonly
affected.
The disease is both more severe and more common in males and cigarette smokers.
Radioactive iodine has been associated with deterioration of the condition.
Steroids are generally reserved for rapidly progressive disease where there is reduction in visual
acuity due to optic nerve compression at the apex of the orbit.

Notes

Family medicine

The most common


Question
1 known form of nutritional deficiency
2 Cause of sore throat
3 Causes of nausea and vomiting

Answer
Iron deficiency
Viral
Viral gastroenteritis
Food poisoning

Type of prevention
-Primary prevention attempts to prevent disease
before it develops.
-Secondary prevention is based on early detection
of an existing pathologic state before it causes
damage to the organism.

Examples
-Avoidance of cigarette smoking.
-Immunization.
-Annual mammography for women older than 45
years.
-Control hypertension and blood lipid levels.
-Pap smear screening.
-Prophylactic aspirin after myocardial infarction.

-Tertiary prevention is based on rehabilitative


interventions or other methods to improve the
clinical evolution of a disease that has already
developed.

Disease
Test or exposure
(+)
(-)
Sensitivity
Percentage of patients with a disease who have a
positive test.
A / (A+C)

(+)

(-)

A
C

B
D

Specificity
Percentage of patients without the disease who
have a negative test.
D / (B+D)

Recommendations
Recommendation for the use of
aspirin in the primary prevention
of cardiovascular disease

Blood pressure recommendations


in patients with diabetes mellitus,
chronic kidney disease, or coronary
artery disease.
There are two recommended
methods for prevention of
pneumonia

Family history

Sex
Age
Potential benefits

Men
4579 years of age
reduction in
myocardial
infarctions

Women
5579 years of age
reduction in
ischemic strokes

The American Heart Association recommends a goal blood


pressure of 130/80 mm Hg or less for the treatment of
hypertension in patients with diabetes mellitus, chronic
kidney disease, or coronary artery disease.
Influenza vaccine- gives yearly to people at increased risk
for complications and to health care workers.
Pneumococcal vaccine- for patients >65 years and for
younger people at high risk (e.g., those with heart disease,
sickle cell disease, pulmonary disease, diabetes, or alcoholic
cirrhosis, or asplenic individuals)
Colon cancer screening
No family history

Age for cancer screening

-From age 50 years

Screening method

-Annually: Fecal occult blood test


or
-Every 5 years: Flexible
sigmoidoscopy or double
contrast barium enemas
or
-Every 10 years: Colonoscopy

Positive family history (one first


degree relative diagnosed with
colorectal cancer or
adenomatous polyps before age
60, or at least two second
degree relatives with colorectal
cancer, are in the highest risk
group)
-At age 40, or 10 years before the
earliest age at which an affected
relative was diagnosed
(whichever comes first) and be
rescreened every 5 years.
Colonoscopy is the preferred
screening method
for this highest-risk group

B1 (Thiamine)

B2 (Riboflavin)
B3 (Niacin)
B6(Pyridoxine)

B12(Cyanocobalamin)

Folic acid
Vitamin A

Vitamin D
Vitamin E

Vitamin K
Vitamin C
Iron

Iodine

Vitamins and Nutritional deficiencies


-Dry Beriberi (Neuropathy)
-Wet Beriberi (High-output cardiac failure)
-Wernicke-Korsakoffs syndrome
Cheilosis (Mouth fissures)
Pellagra (Diarrhea, Dementia, Dermatitis)
-Dermatitis
-Stomatitis
-Glossitis
-Peripheral neuropathy
-Megaloblastic anemia
-Posterior spinal column neuropathy
-Peripheral Neuropathy
-Atrophic Glossitis
-Neural tube defect
-Megaloblastic anemia
-Night blindness
-Corneal ulceration
-Acne
-Bitots spots
-Frequent respiratory infections
-Rickets
-Osteomalacia
-Opthalmoplegia
-Sensory and motor peripheral neuropathy
-Fragile RBCs
Clotting deficiency
Scurvy (poor wound healing, bleeding gum, purpura, easy bruising)
-Anemia
- koilonychias (spoon-shaped nails)
-Brittle hair
-Loss of appetite
-pruritis
-Glossitis
-Cheilosis
-Hypothyroidism
-Cretinism
-Goiter

Quick revision

Question
Anticholinergic drugs (e.g., Benzotropine) are
contraindicated in

Female Athlete triad

Treatment options of nonspecific chronic back pain

The organisms causing Non-Gonococcal Urethritis

pharmacological prevention of acute mountain


sickness

Incidence definition

Prevalence definition

P value

Answer
Angle-Closure Glaucoma
Urinary retention
GI obstruction
Amenorrhea
Disorder eating
Osteoporosis
Analgesics (acetaminophen,
tramadol, NSAIDs)
Multidisciplinary
rehabilitation
Acupuncture
Chlamydia trachomatis 40%
Ureaplasma urealyticum
Trichomonas vaginalis
Candidiasis
Herpes Simplex

Acetazolamide

Contraindicated
in patients with
a sulfa allergy
Dexamethasone An effective
prophylactic
and treatment
agent

Number of new disease cases per


population at risk measured over a given
time interval.
Number of current cases per population
at risk measured over a given time
interval.
The measured probability of a
finding occurring by chance
alone, given the null hypothesis
is actually true.
Significant P value is < 0.05

Laboratory markers, diagnostic and screening methods


Question
Answer
The gold standard non-invasive test for
Serum Ferritin
diagnosing Iron deficiency anemia and the
best parameter to assess response to Iron
therapy
The most effective method of screening for Colonoscopy
colon cancer
The most effective method to reduce
Screening mammography
breast cancer mortality for women
between 40-49 years of age

Stages in the Family-Life Cycle


"Launching family" in family life cycle Starts when the first child- not necessarily the eldest leaves the home and ends when the last child leaves the home.
In "The new couple "stage in family life cycle, the Couple must build a base of shared value such
as who will go to the grocery shopping or preparing the food.
The highest incidence of divorce comes at Child rearing stage.

Drug
Hydroxychloroquine
Chloramphenicol
Nitrofurantoin
Gentamicin
Tetracycline
Clindamycin
Rifampicin

Side effects
Retinal toxicity
Aplastic anemia
Peripheral neuropathy
Ototoxicity
Nephrotoxicity
Dental staining of fetus if administered to the mother
Pseudomembranous colitis
Red discoloration of urine
Leukopenia
Thrombocytopenia

Notes

Emergency medicine

Toxicology
Poison or Medication
Acetaminophen
Cholinesterase inhibitors
Quinidine or Tricyclic anti-depressants
Iron
Digoxin
Methanol/ethylene glycol
Benzodiazepines
Beta blockers
Lead
Cooper or gold
Opioids
Carbone monoxide
Muscarinic blockers

Antidote
Acetylcysteine
Atropine, Pralidoxime
Sodium bicarbonate
Deferoxamine
Digoxin immune Fab
Ethanol
Flumazenil
Glucagon
EDTA/Succimer
Penicillamine
Naloxone
Oxygen
Physostigmine

Notes

Psychiatry

Quick review
Question
Anorexia nervosa( 15% below normal weight ) is
associated with

Antidepressants complications
(Food and drugs interactions)

Side effects of typical antipsychotic medications

Side effects of atypical antipsychotic medications


The best initial treatment for major depressive
disorder, anxiety and social phobia
Electroconvulsive therapy (ECT) contraindications
The most appropriate first line management of
severe behavioral symptoms with psychotic
features in nursing home patients
The only agents that have been approved by the
FDA for the treatment of Post traumatic stress
disorder
The best for rapid relief of symptoms in severely
distressed patient with panic disorder

Answer
Amenorrhea, Osteoporosis, Hypokalemia,
Raised plasma cortisol with loss of diurnal
variation, Hypoglycemia, Erosion of tooth
enamel, Bradycardia, Hypotension, Cardiac
arrhythmias, Edema, Hypothermia, Low
gonadotrophins and Low triiodothyronine
Overconsumption of tyramine-containing
foods (Cheese) in a patient receiving
MAOIs (Phenelzine) may result in
hypertensive crisis.
Opiods (Meperidine and
dextromethorphan) may cause serotonin
syndrome in patients who are chronically
taking SSRIs or MAOIs.
Extrapyramidal symptoms, Hyperprolactenemia,
anticholinergic effects, seizures and neuroleptic
malignant syndrome.
Weight gain, DM type 2, QTc prolongation and
agranulocytosis (clozapine)
Selective serotonin reuptake inhibitor
There are no absolute contraindications to
electroconvulsive therapy (ECT).
Atypical antipsychotics e.g. risperidone

Sertaline and Paroxetine

Alprazolam (xanax)

First-line treatment of Attention deficit hyperactivity


disorder
The Mini-Mental State Examination (MMSE) goal
Tourettes syndrome association

Stimulant medication (e.g. Methylphenidate).

The most effective treatment of bulimia nervosa


A successful approach for treatment of the adolescent

Fluxetine

with anorexia nervosa

(MMSE) tests for cognitive function


Tourettes syndrome is often associated with psychiatric
comorbidities, mainly attention-deficit/hyperactivity
disorder and obsessive-compulsive disorder.
Family-based treatment for the adolescent with
anorexia nervosa has been found to provide successful
results.

Somatoform and Factitious disorders


Unintentionally
Concept

A patient with
somatoform disorder
experiences psychiatric
stress and expresses it
through physical
symptoms. Patients do
not do so on purpose.
Major somatoform disorders
Somatization disorder
The patient has multiple
(Briquet's syndrome)
different complaints in
multiple different organ
system over many years
and has had extensive
work-ups in the past.
Conversion disorder
The patient has an
(Hysterical neurosis)
obvious precipitating
factor (e.g., fight with
boyfriend), then
develops unexplainable
neurologic symptoms
(e.g., blindness, stockingglove numbness).
Hypochondriasis
The patient continues to
believe that he or she
has a serious disease
despite extensive
negative work-up.
Body dysmorphic
The patient is
disorder
preoccupied with an
imagined physical defect;
for example, a teenager
who thinks that his or her
nose is too big when it is
normal size.
Treatment
Treat all somatoform
disorders with frequent
return visits to the clinic
and/or psychotherapy.
Screen for and treat any
coexisting depression.

Intentionally
-Factitious disorders:
Patients intentionally
create an illness or
symptoms (e.g., they
inject insulin to create
hypoglycemia) and
subject themselves to
procedures in order to
assume the role of a
patient (no financial or
other secondary gain).
-Mnchausen syndrome
by proxy:
A behavior pattern in
which a caregiver
fabricates, exaggerates,
or induces health
problems in those who
are in their care.

Malingering:
Patients intentionally
create their illness for
secondary gain (e.g.,
money, release from
work or jail)

Antipsychotics side effects


Acute dystonia

Akathisia

Parkinsonism

Tardive Dyskinesia

Malignant syndrome

Description
Occurs within hours of medication (most common with highpotency IM use).
Torticollis, jaw dislocation and tongue protrusion.
Usually disappears eventually (Tolerance).
May occur at any time.
Feeling of muscular discomfort.
Relentless movements.
Occurs within weeks to months.
Muscle stiffness: Cogwheel rigidity.
Shuffling, drooling.
Usually disappears eventually (Tolerance).
Occurs after many months of treatment.
Choreoathetosis, tongue protrusion and lateral movements of jaw.
Most mild cases eventually remit but more severe ones are often
irreversible.
High fever, heart rate and blood pressure.
Muscle rigidity.

Antidepressants
Serotonin selective reuptake inhibitors

Tricyclic antidepressants

Monoamine oxidase inhibitors

Lithium

Side effects
-Decreased libido
-Insomnia
-Restlessness
-Decreased appetite
-Blurred vision
-Dry mouth
-Constipation
-Urinary retention
-Sedation
-ECG changes; depressed ST and prolonged PQ
-Orthostatic hypotension
-Weight gain
-Edema
-Tremors
-Nausea, vomiting, diarrhea
-Confusion
-convulsions
-Ataxia

Mild

Moderate

Severe

Tricyclic compounds overdose( anticholinergic effects)


Presentation
Management
-Midyriasis
-Main goal: Maintenance of serum PH at 7.50 (alkalotic
-Impaired sweating
state) by:
-Constipation
1-Hyperventolation
-Flushed skin
2-IV bicarbonate
-Tachycardia
3-Gastric aspiration and lavage should be performed to
-Muscle twitching
eliminate unabsorbed drug if more than 750 mg of TCA
has been taken.
-Cardiac arrhythmias (ventricular
tachyarrhythmia and bradycardia)
-prolongation of the QRS complex (>0.1)
-Seizure
-Severe hypotension
-Coma

Dysthymic disorder
Characterized by depressed mood for at least 2
years in addition to at least two of the following:
change in appetite, alteration in sleep, low energy,
low self-esteem, poor concentration, or feelings of
hopelessness. There must be no history of a manic
or hypomanic episode, substance abuse, a chronic
psychotic disorder, or an organic cause.

Cyclothymia
Chronic, but less extreme, form of bipolar disorder
that consists of short periods of mild depression
alternating with short periods of hypomania. The
onset of each phase is separated by short periods
of normal mood. This diagnosis is excluded if the
patient has had either a manic episode or a major
depressive episode.

Notes

Dermatology and Immunology

The most common


1
2
3
4
5
6
7

Question
Cause of Erythema multiforme
Skin cancer
Cause of death due to skin cancer
(Most aggressive form of skin cancer)
Type of alopecia
Type of psoriasis
Causative organism of cellulitis and erysipelas
Causative organism of folliculitis is

Answer
Herpes simplex virus
Basal cell carcinoma
Melanoma
Androgenetic baldness
Stable plaque psoriasis
Group A streptococcus
Staphylococcus aureus

Erythrasma
Definition

Causative
organism

Distribution

Diagnosis

Treatment

-A skin disease
that causes small
reddish-brown
macules that
may coalesce
into larger
patches with
sharp borders.
-It is prevalent
among diabetics
and the obese,
and in warm
climates; it is
worsened by
wearing
occlusive
clothing.

Gram-positive
bacterium
Corynebacterium
minutissimum

In intertriginous
areas;
-Interdigital web
spaces
-Gluteal crease
-Inguinal area
-Axilla
-Inframammary

-Wood's Lamp:
Fluorescence
coral red.
-Gram Stain:
Gram Positive
rod with long
filaments.

Topical or oral
erythromycin

Multiple seborrhoeic warts

May occur in any area where there are


pilosebaceous follicles, but are most
commonly found on the face and
trunk.
They are non-infective.
They are best removed by curettage or
cryotherapy.
They have a recognized association
with internal malignancy (Leser-Trelat
sign).

Cowden disease (multiple hamartoma


syndrome)
Autosomal dominant condition
Characterized by: multiple
trichilemmomas, buccal papillomatosis,
lipomas and angiomas.
Predisposition to carcinoma of the
breast and thyroid.

Type I hypersensitivity (or


immediate hypersensitivity)
Type II hypersensitivity
Type III hypersensitivity

Type IV hypersensitivity (or


delayed hypersensitivity)

Types of Hypersensitivity Reaction


IgE-mediated
IgG- (or IgM- ) and Cytotoxic cell
mediated
Antigen-Antibody complex

T-cell mediated

-Anaphylaxis
-Asthma
-Good pastures disease
- Pemphigus vulgaris
-SLE
-Arthus reaction
-Serum sickness
-Allergic contact dermatitis
-Tuberculosis
-Transplant rejection

Quick revision
Question
Actinic keratoses definition

Cutaneous leishmaniasis definition

The most likely metal to cause contact


dermatitis
The most likely diagnosis of frequent
episodes of swollen lips and difficulty
breathing
The scabies mite is predominantly
transmitted by
Tenia versicolor can be diagnosed by
demonstration of
First-line treatment of lichen planus

Answer
Scaly lesions that develop on sun-exposed skin,
and are believed to be carcinoma in situ. While
most actinic keratoses spontaneously regress,
others progress to Squamous cell cancers.
Caused by an intracellular parasite transmitted
by the bite of small sand flies. Lesions develop
gradually, and are often misdiagnosed as
folliculitis.
Nickel
Angioedema

Personal contact
Hyphe
High-potency topical corticosteroids such as
Clobetasol.

Notes

Objective Structured Clinical Examination

Diagnosis
Atrial fibrillation

Diagnosis
Atrial flutter

Diagnosis
Supraventricular tachycardia

Diagnosis
Ventricular tachycardia

Diagnosis
Ventricular fibrillation

Diagnosis
First degree heart block

Diagnosis
Mobitz type 1

Diagnosis
Mobitz type 2

Diagnosis
Third-degree atrioventricular block

Diagnosis
Anterolateral ST elevation MI

Diagnosis
Inferior wall MI

Diagnosis
Hyperkalemia

Diagnosis
Erythema nodosum

Diagnosis
Erythema multiforme

Audiogram interpretation
Normal hearing level

Audiogram interpretation
Sensory neural hearing loss

Audiogram interpretation
Conductive hearing loss

Audiogram interpretation

Mixed hearing loss

CTG interpretation

Acceleration

CTG interpretation

Early decelerations

CTG interpretation

Late decelerations

CTG interpretation

Variable decelerations

Blood Bottles Guide

Name this instrument


Intravenous cannula

Name this instrument


Endotracheal tube

Name these instruments


Mechanical heart valves

Name this instrument


Nasogastric tube

Name this instrument


Grommet Tube

Name this instrument


Ayre spatula

Name this instrument


Sims Speculum

Name this instrument


Cussco Speculum

Name this instrument


Hegar's dilator

Name this instrument


Amniohook

Name this instrument


Ventouse

Name this instrument


Double J Stents

Name this instrument


Sangstaken-Blakemore tube

Diagnosis
Management

Right Pneumothorax
Chest tube insertion

Describtion and most likely diagnosis


Right middle lobe infiltration suggestive of pneumonia

Describtion

AP view of Right arm showing a transverse


Mid Shaft humeral fracture with angulation

Management

1-U shaped splint ( conservative)


2- Open reduction internal fixation if
conservative failed

Complication

Radial nerve injury


wrist drop

Diagnosis
Tests

DDH developmental dysplasia of the hip


Barlow's Test: dislocation
Ortolani test: relocation

Treatment

pavlik harness

Type of radiological study

Finding

Ultrasound

Gallbladder stone

Diagnosis
Diaphragmatic hernia

Diagnosis
Cardiac tamponade

Diagnosis
Toxic megacolon
Can be a complication of IBD or Pseudomembranous colitis

Diagnosis
Pleural effusion

Diagnosis

Staghorn stone

Type

Struvite ( infectious)

Risk Factors

Anatomical malformation of the


urinary tract
UTI

Management

Admission
I.V analgesics
I.V hydration
I.V antibiotics
NPO
Percutaneous nephrolithotomy

A child developed these lesions after being taken to a farm

Diagnosis

Urticaria

Common
causes

Nuts, Eggs, strawberries


Drugs: e.g. (Aspirin, NSAIDS, ACEI)
Infections
Insect bite

Treatment

Make sure the airway is clear (make sure


there is no airway swelling due to
angioedema)
Antihistamine
If severe with angioedema give adrenaline
immediately.

Diagnosis

Varicose veins

Pathophysiology Venous valves incompetency


Treatment

1-conservative :
Compression stockings
2- surgical :

Sclerotherapy

Stripping and ligation

Describe the
abnormalities.

Protruded tongue
Deviation of the tongue to the left side
Atrophy and fasciculation on the left side

What is your Diagnosis?

Left lower motor neuron hypoglossal nerve palsy (12)

What are the possible


causes?

Tumor
Trauma to the base of the skull
Multiple Sclerosis
Infection

Diagnostic test

MRI

Diagnosis
Right lower motor neuron facial
nerve palsy (bells palsy)

Causes
Infections : e.g. HSV , HZV , Lyme disease
Tumors
Autoimmune diseases

Central venous catheter


Indications

Administration of fluid & drugs


Monitoring
Hemodialysis

Complications

Pneumothorax
Hematoma
Infection
Thrombus
Arrhythmias

Chalazion
Inflammation of a blocked meibomian
gland

Stye
An infection of the sebaceous glands of
Zeis at the base of the eyelashes, or an
infection of the apocrine sweat glands of
Moll

Diagnosis

Papilledema

Causes

Tumor
Malignant hypertension
Intracranial hemorrhage
Pseudotumor cerebri

Complication

Optic atrophy
Visual loss

Describtion

Plain radiograph of the right hip


joint showing comminuted displaced
angulated proximal femur fracture

Complications

Hemorrhage
Pulmonary embolism/ fat embolism
Hematoma

Management

Open reduction internal fixation,


Intramedullary nail, physiotherapy

Describtion

X-ray showing:
Proximal humerus fracture with
displacement (surgical neck fracture)

Associated nerve injury

Axillary nerve

Complications

Weakness of the deltoid muscle


Open fracture
Malunion

Describtion

X-ray showing:
Flattening of the femoral head
Irregular sclerosis
Narrow cartilage space
Lucency of the underlying bone

Diagnosis

Avascular necrosis of the femoral head

Most common cause in Bahrain

Sickle cell disease

Treatment

Hydration
Analgesics
Exchange transfusion
Core decompression of the head

Describtion

Bilateral diabetic ulcers of the feet located on the plantar


surface of both feet on weight bearing area , punched out
appearance, with callus formation around its borders

Diagnosis

Diabetic foot ulcer

Management

Education
Tight glycemic control
Good hygiene
Debridement
Dressing
Antibiotics

This patient presented with abdominal pain and


hematuria

What is your diagnosis?

HenochSchnlein purpura

What are the lab findings?

INCREASE IN:
Urea
Creatinine
CRP
ESR
IgA
Platelets

How to confirm your


diagnosis?

Biopsy is the most accurate test and it will show


leukocytoclastic vasculitis (small vessels)

Treatment

Conservative

What is the indication of


steroid therapy?

Steroids are indicated in severe abdominal pain or


progressive renal insufficiency

What is the name of this appearance?

Bamboo spine

What is the diagnosis?

Ankylosing spondylitis

What is the characteristic antigen?

HLA- B27 antigen

Treatment

NSAIDS, Methotrexate, Infliximab

A pregnant model with transverse lie

How to manage her delivery?

External cephalic version by 37 weeks ,


Success rate is 50%
Prepare for LSCS
Blood matching and grouping
CTG , Biophysical profile

complications

-Prolapsed cord
-Uterine rupture
-Antepartum hemorrhage

Mention 4 differential diagnosis of


not itchy hypopigmented lesions

Pitryasis alba
Pitryasis versicolor
Vitilligo
Mycosis fungoides

Most likely diagnosis

Pitryasis Alba

Treatment

Education
Avoid sun exposure
Non-soap cleanser
Topical steroids
PUVA therapy

Gastrostomy tube
Indications

Enteral feeding, stroke, decompression ,


esophageal stricture, recurrent aspiration

Complications

skin irritation, infection

Describtion

Plain CXR showing cardiomegaly,


alveolar edema, haziness of vascular
margins

Differential diagnosis

Cardiogenic pulmonary edema


Congestive heart failure

Foleys catheter

Indications

Urinary retention
Comatose patient
Anaesthesia or sedation
Urinary obstruction

Complications

UTI
Urethral injury

Diagnosis

Nephrotic syndrome

Diagnostic criteria

Proteinuria > 3.5 g/day


Generalized edema
Hyperlipidemia
Hypoalbuminemia
Hypercoagulation

Diagnosis

Scarlet fever

Causative organism

Streptococcus pyogenes (group A)

Complications

Sepsis
Glomerulonephritis
Rheumatic fever
Erythema nodosum

Treatment

Amoxicillin

Diagnosis
Gouty tophi (nodules)

Describtion

AP view of Chest x-ray showing bilateral


pulmonary infiltrates in the upper lobes
with cavity formation

Most likely diagnosis

pulmonary T.B

Treatment

Isoniazid , Rifampicin , Ethambutol,


Pyrazinamide

Ileostomy
Indications Diseases of the large intestine which may require surgical
removal including :
Crohn's disease
Ulcerative colitis
Familial adenomatous polyposis
Total colonic Hirschsprung's disease

Diagnosis

Air under diaphragm

Causes

Perforated duodenal ulcer


Perforated peptic ulcer
Ruptured diverticulum
Penetrating trauma
After laprotomy or laparoscopy

Intra uterine contraceptive device


Types

1-copper
2-Hormonal

Absolute contraindications

-Current pregnancy
-Undiagnosed abnormal vaginal bleeding
-Acute infection

Relative contraindications

-Previous ectopic
-History of STD

Complication

-Increased bleeding
-Expulsion
-Uterine perforation
-PID

Diagnosis
Tympanic membrane perforation

Causes
-Trauma
-Otitis media

Diagnosis

1-Tinea capitis (scalp)


2- Tinea pedis (Athletes foot)
3- Tinea corporis (body)

Causative organism

Dermatophytes

Treatment

Topical antifungal e.g. meconazole

Diagnosis
Common GIT problems
Radiological sign

Down Syndrome
Duodenal atresia
Double bubble

Tracheostomy tube
Indications

-Severe facial trauma causing obstruction


-Edema from trauma, burns, infection, or anaphylaxis
-Supraglottic or glottic pathologic condition (e.g., infection,
neoplasm, bilateral vocal cord paralysis)
-Congenital anomaly (e.g., laryngeal hypoplasia, vascular
web)
-Severe sleep apnea not amendable to continuous positive
airway pressure devices or other less invasive surgery

Diagnosis

Mucopolysaccharidosis

Genetics

-Absence or malfunctioning of lysosomal


enzymes needed to break down molecules called
glycosaminoglycans (mucopolysaccharides).
-Hunter syndrome is X-linked.
-Hurler syndrome is autosomal recessive.

Physical
features

-A flat nasal bridge


-Thick lips
-Enlarged mouth and tongue
-Short stature with disproportionately Short trunk
(dwarfism)
-Excessive body hair growth
-Short and often claw-like hands

Complications

-Hepatosplenomegaly
-Progressive joint stiffness, and carpal tunnel
syndrome
-Recurring respiratory infections are common
-Obstructive airway disease and obstructive sleep
apnea
-Hernias

Diagnosis
Eye findings

Complications

Pancoast tumor
-Ptosis
-Miosis
-Anhydrosis
-Horners syndrome(compression of sympathetic ganglion)
-SVC obstruction
-Compression of subclavian artery
-Painful shoulder
-Pulmonary complications: hemoptysis , dyspnea
-Invasion of the brachial plexus.

Findings

PA view of a plain chest X-ray showing dextrocardia,


right sided gastric bubble, and the left hemidiaphragm is
elevated more than the right suggesting that the liver is
present on the left side.

Diagnosis

Kartagener syndrome
( situs inversus totalis)
(Immotile cilia syndrome)

Complications

-Infertility
-Infection ( bronchoectasois )(recurrent chest infections)
-Icteric
-Short stature
-Sinusitis
-Suppurative otitis media
-Splenomegaly

Name the skin change


Peau d'orange

Pedigree

What is the pattern of


inheritance?

Mention 3 examples of
conditions inherited by
this pattern?

Autosomal dominant

Huntington's disease
Marfans syndrome
Achondroplasia

Describtion

A peripheral blood smear showing


Ring-forms of Plasmodium falciparum

Complications

Respiratory distress
Renal failure
Cerebral malaria
Antimalarial agents e.g. Quinine

Treatment

Describtion

A peripheral blood smear Showing Hyper


segmented neutrophils

Diagnosis

Megaloblastic anemia

Causes

Vitamin B12 deficiency


Folate deficiency
Combined deficiency
Drugs : e.g. Methotrexate & Phenytoin

Diagnosis

Fifth disease
(slapped cheek appearance)

Causative organism

Parvo virus B19

Life threatening complications

Bone marrow failure,


secondary infection and sepsis.

You might also like